|

UPSC IAS Civil Services Preliminary (Prelims) Exam – 2023 Question Paper with Answer key and Explanation

The UPSC (Union Public Service Commission) conducted the UPSC Prelims 2023 examination on 28th May 2023. Aspirants who appeared for the exam can now refer to the question paper of General Studies along with the answer key and explanations. The official PT Answer Key 2023 will be released soon by the UPSC, allowing candidates to calculate their estimated scores. The answer key will be available in PDF format for all sets – A, B, C, and D.

Using the UPSC IAS Answer Key, candidates can compute their approximate scores for the UPSC CSE 2023 Prelims. The marking criteria for Paper I awards 2 marks for each correct answer, while Paper II allots 2.5 marks for each correct response. However, it’s important to note that for each incorrect answer, 1/3rd of the mark allotted to a question is deducted.

By referring to the UPSC IAS Prelims Answer Key 2023 with explanations, students can assess their correct and incorrect responses, enabling them to calculate their marks for the Prelims examination. It’s crucial to remember that while the Prelims exam is the first step of the selection process, the marks obtained in this stage will not be considered when preparing the Final Merit List. The final selection for the prestigious IAS (Indian Administrative Service) post depends on the candidate’s performance in the UPSC Mains exam and Interview.

UPSC 2023 Prelims Question Paper – GS I and CSAT

UPSC General Studies-1 2023 Question Paper
UPSC CSE Prelims 2023 – SET ADownload PDF
UPSC CSE Prelims 2023 – SET BDownload PDF
UPSC CSE Prelims 2023 – SET CDownload PDF
UPSC CSE Prelims 2023 – SET DDownload PDF
UPSC General Studies-2 2023 CSAT Question Paper
UPSC CSE Prelims 2023 – CSATDownload PDF

Prelims 2023 General Studies Paper 1 – Questions with Answer Key and Explanation

1. Consider the following statements :

  1. Jhelum River passes through Wular Lake.
  2. Krishna River directly feeds Kolleru Lake.
  3. Meandering of Gandak River formed Kanwar Lake.

How many of the statements given above are correct?

(a) Only one
(b) Only two
(c) All three
(d) None

Answer – The correct answer is B, i.e Only two.

Statement 1: The Jhelum, a significant tributary of the Indus, originates from a spring located at Verinag, situated at the base of the Pir Panjal range in the southeastern part of the Kashmir Valley. It passes through Srinagar and the Wular Lake before entering Pakistan through a deep narrow gorge. It joins the Chenab River near Jhang in Pakistan.

Statement 2: This statement is incorrect. Although Kolleru Lake lies between the deltas of the Godavari and Krishna rivers, the Krishna River does not directly contribute to the water supply of Kolleru Lake. Instead, the lake is directly fed by the seasonal Budameru and Tammileru streams.

Statement 3: This statement is correct. Kanwar Lake is a residual oxbow lake that formed as a result of the meandering of the Gandak River.

Source 1 – https://ncert.nic.in/ncerts/l/kegy103.pdf

Source 2 – https://www.academia.edu/37482845/KANWAR_LAKE_A_DYING_WETLAND_ECOSYSTEM

2. Consider the following pairs:

Port – Well known as

  1. Kamarajar Port – First major port in India registered as a company
  2. Mundra Port – Largest privately owned port in India
  3. Visakhapatnam Port – Largest container port in India

How many of the above pairs are correctly matched?

(a) Only one pair
(b) Only two pairs
(c) All three pairs
(d) None of the pairs

  1. The correct answer is B, i.e. Only two.

Statement 1: This statement is correct. Kamarajar Port, situated on the Coromandel Coast approximately 24 km north of Chennai Port, Chennai, is the 12th major port of India and the first port in India that operates as a public company.

Statement 2: This statement is correct. Mundra Port holds the distinction of being the largest privately owned port in India.

Statement 3: This statement is not correct. JNPT (Jawaharlal Nehru Port Trust) is one of the largest container ports in India.

Source – Businessline

3. Consider the following trees:

  1. Jackfruit (Artocarpus heterophyllus)
  2. Mahua (Madhuca indica)
  3. Teak (Tectona grandis)

How many of the above are deciduous trees?

(a) Only one
(b) Only two
(c) All three
(d) None

Answer – The correct answer is B, i.e Only two.

Statement 1: This statement is incorrect. The jackfruit tree can be found in evergreen forests, so it is not incorrect.

Statement 2: Both mahua and teak are deciduous trees.

Source – https://pfaf.org/user/Plant.aspx?LatinName=Artocarpus+heterophyllus

4. Consider the following statements

  1. India has more arable area than China.
  2. The proportion of irrigated area is more in India as compared to China.
  3. The average productivity per hectare in Indian agriculture is higher than that in China.

How many of the above statements are correct?

(a) Only one
(b) Only two
(c) All three
(d) None

Answer – The correct answer is B, i.e Only two.

Statement 1: This statement is correct. India and China, being the most populous countries in the world, have limited arable land. China has approximately 120 million hectares (mha) of arable land, while India has 156 mha.

The challenge facing both countries is to produce enough food, fodder, and fiber to meet the needs of their populations.

Both countries have embraced modern agricultural technologies, starting with high yield variety (HYV) seeds in the mid-1960s. They have also increased irrigation coverage and employed more chemical fertilizers to enhance food production on limited land.

Statement 2: This statement is correct. China’s irrigation coverage accounts for 41 percent of its cultivated area, while India’s irrigation coverage is 48 percent.

As a result of this irrigation, China’s total sown area is 166 mha, whereas India’s gross cropped area is 198 mha. However, even though China has less land under cultivation, its agricultural output is valued at $1,367 billion, which is more than three times that of India’s agricultural output valued at $407 billion. Hence, Statement 3 is NOT correct.

Source – https://indianexpress.com/article/opinion/columns/india-china-agriculture-output-demands-akis-6090647/

5. Which one of the following is the best example of repeated falls in sea level, giving rise to present-day extensive marshland?

(a) Bhitarkanika Mangroves
(b) Marakkanam Salt Pans
(c) Naupada Swamp
(d) Rann of Kutch

Answer – The correct answer is D, i.e Rann of Kutch.

Among the options provided, the Rann of Kutch in the state of Gujarat, India, stands out as an example of how repeated declines in sea level have given rise to vast marshlands that exist today.

Situated in western India, the Rann of Kutch encompasses a vast expanse of salt marshland featuring seasonal marshes, salt flats, and shallow wetlands. This unique ecosystem undergoes significant variations in sea level due to geological and climatic factors.

Over time, the Rann of Kutch has witnessed a dynamic history of marine incursions and retreats, characterized by periods of sea advancement and recession. These recurring episodes of falling sea levels have played a crucial role in shaping the expansive marshland we observe today. During the wet season, the Rann of Kutch submerges underwater, transforming into a shallow marshland. In stark contrast, during the dry season, the water evaporates, leaving behind salt flats and desolate terrain.

6. Ilmenite and rutile, abundantly available in certain coastal tracts of India, are rich sources of which one of the following?

(a) Aluminium
(b) Copper
(c) Iron
(d) Titanium

Answer – The correct answer is D, i.e Titanium.

  • Ilmenite and rutile, which are abundantly available in certain coastal tracts of India, are rich sources of titanium.
  • These minerals contain significant amounts of titanium dioxide, a compound widely used in various industries.
  • Titanium dioxide is particularly valued for its high refractive index, which makes it an essential ingredient in the production of pigments, coatings, and paints. It is also used in the manufacturing of ceramics, plastics, sunscreen, and other products where its excellent light-scattering and UV-absorbing properties are desired.

Source – https://ibm.gov.in/writereaddata/files/08172015131610Ilmenite%20and%20Rutile.pdf

7. About three-fourths of world’s cobalt, a metal required for the manufacture of batteries for electric motor vehicles, is produced by

(a) Argentina
(b) Botswana
(c) the Democratic Republic of the Congo
(d) Kazakhstan

Answer – The correct answer is C, i.e the Democratic Republic of the Congo.

The Democratic Republic of Congo (DRC) is responsible for producing over 70 percent of the global cobalt supply. Within the DRC, approximately 15 to 30 percent of the cobalt production comes from artisanal and small-scale mining (ASM).

  • Cobalt is a crucial metal used in the production of various industries, including batteries for electric vehicles, electronic devices, aerospace applications, and renewable energy systems.
  • Within the DRC, a significant portion of cobalt production, estimated to be between 15 to 30 percent, is attributed to artisanal and small-scale mining (ASM). ASM refers to mining activities carried out by individuals or small groups using basic tools and techniques, often in informal or unregulated settings.
  • Artisanal cobalt mining in the DRC involves a labor-intensive process where miners manually extract cobalt-rich ores from the ground. These ores are then processed through rudimentary means to separate cobalt from other minerals. ASM operations in the DRC often face challenges related to inadequate safety measures, environmental impact, and social issues such as child labor and poor working conditions.
  • Due to its dominance in cobalt production, the DRC’s mining sector plays a significant role in the global cobalt supply chain. Efforts are being made by various stakeholders, including international organizations, mining companies, and the DRC government, to address sustainability, transparency, and ethical concerns associated with cobalt mining in the country.

Source – https://www.cfr.org/blog/why-cobalt-mining-drc-needs-urgent-attention

8. Which one of the following is a part of the Congo Basin?

(a) Cameroon
(b) Nigeria
(c) South Sudan
(d) Uganda

Answer – The correct answer is A, i.e Cameroon.

The Congo Basin encompasses six countries, namely Cameroon, Central African Republic, Democratic Republic of the Congo, Republic of the Congo, Equatorial Guinea, and Gabon.

Source – https://www.worldwildlife.org/places/congo-basin

9. Consider the following statements:

  1. Amarkantak Hills are at the confluence of Vindhya and Sahyadri Ranges.
  2. Biligirirangan Hills constitute the easternmost part of Satpura Range.
  3. Seshachalam Hills constitute the southernmost part of Western Ghats.

How many of the statements given above are correct?

(a) Only one
(b) Only two
(c) All three
(d) None

Answer – The correct answer is D, i.e None.

The Narmada River originates on the western flank of the Amarkantak plateau at an elevation of approximately 1,057 meters. It flows through a rift valley located between the Satpura Range in the south and the Vindhyan Range. Therefore, Statement 1 is incorrect.

The Biligirirangan Hills are indeed situated in southeastern Karnataka; however, the statement erroneously mentions the Satpura Range, which is located in the Madhya Pradesh region. Hence, Statement 2 is incorrect.

The Seshachalam Hills are part of the Eastern Ghats mountain range. Consequently, Statement 3 is also incorrect.

Source – https://ncert.nic.in/ncerts/l/kegy103.pdf

10. With reference to India’s projects on connectivity, consider the following statements:

  1. East-West Corridor under Golden Quadrilateral Project Dibrugarh and Surat. connects
  2. Trilateral Highway connects Moreh in Manipur and Chiang Mai in Thailand via Myanmar.
  3. Bangladesh-China-India-Myanmar Economic Corridor connects Varanasi in Uttar Pradesh with Kunming in China.

How many of the above statements are correct?

(a) Only one
(b) Only two
(c) All three
(d) None

Answer – The correct answer is D, i.e None.

Statement 1 is incorrect. The East-West corridor of the Golden Quadrilateral superhighway connects Silchar and Porbandar.

Statement 2 is incorrect. The India-Myanmar-Thailand trilateral highway, also known as the Asian trilateral highway, is indeed an under-construction international highway. However, it will connect India (Moreh, Manipur) and Thailand (Mae Sot) via Myanmar.

Statement 3 is incorrect. The BCIM (Bangladesh-China-India-Myanmar) economic corridor aims to connect Kolkata with Kunming, the capital of China’s Yunnan province, not Silchar and Porbandar.

Source – https://www.thehindu.com/news/international/bangladesh-china-india-myanmar-bcim-economic-corridor-no-longer-listed-under-bri-umbrella/article26971613.ece

11. Consider the following statements

Statement-I:
India, despite having uranium deposits, depends on coal for most of its electricity production.

Statement-II:
Uranium, enriched to the extent of at least 60%, is required for the production of electricity.
Which one of the following is correct in respect of the above statements?

(a) Both Statement-I and Statement-II are correct and Statement-II is the correct explanation for Statement-I
(b) Both Statement-I and Statement-II are correct and Statement-II is not the correct explanation for Statement-I
(c) Statement-I is correct but Statement-II is incorrect
(d) Statement-I is incorrect but Statement-II is correct

Answer – The correct option is C.

Statement 1 is correct: Coal holds significant importance in India as the country’s most vital and abundant fossil fuel. It satisfies approximately 55% of the nation’s energy requirements.

Statement 2 is incorrect: Most reactors do not require uranium to be enriched from 0.7% to 3-5% U-235 in their fuel for electricity production. While some reactors, such as light-water reactors, utilize enriched uranium, there are various types of reactors that employ different fuel compositions and enrichment levels.

Source 1 – Ministry of Coal

Source 2 – World Nuclear Organisation

12. Consider the following statements :

Statement-I:
Marsupials are not naturally found in India.

Statement-II:
Marsupials can thrive only in montane grasslands with no predators.

Which one of the following is correct in respect of the above statements?

(a) Both Statement-I and Statement-II are correct and Statement-II is the correct explanation for Statement-I
(b) Both Statement-I and Statement-II are correct and Statement-II is not the correct explanation for Statement-I
(c) Statement-I is correct but Statement-II is incorrect
(d) Statement-I is incorrect but Statement-II is correct

Answer – The correct answer is C.

Statement 1 is correct: Marsupials are endemic to Australasia, the Americas, and Wallacia.

Statement 2 is incorrect: Marsupials are also well-suited to thrive in dry grasslands. They are more commonly found in a variety of habitats including forests, woodlands, and even some deserts, but they generally prefer areas with sufficient vegetation and resources for their unique reproductive and parenting strategies.

Source – https://www.britannica.com/animal/marsupial

13. Invasive Species Specialist Group’ (that develops Global Invasive Species Database) belongs to which one of the following organizations?

(a) The International Union Conservation of Nature
(b) The United Nations Environment Programme
(c) The United Nations World Commission for Environment and Development
(d) The World Wide Fund for Nature

Answer – The correct answer is A, i.e IUCN.

The Invasive Species Specialist Group (ISSG) is a worldwide network comprising scientific and policy experts focused on invasive species. It operates under the umbrella of the Species Survival Commission (SSC) within the International Union for Conservation of Nature (IUCN).

Source – https://www.eea.europa.eu/data-and-maps/data-providers-and-partners/invasive-species-specialist-group-issg

14. Consider the following fauna:

  1. Lion-tailed Macaque
  2. Malabar Civet
  3. Sambar Deer

How many of the above are generally nocturnal or most active after sunset?

(a) Only one
(b) Only two
(c) All three
(d) None

Answer – The correct answer is B, i.e Only two.

Statement 1 is incorrect: The lion-tailed macaque, also known as the wanderoo, is primarily arboreal and thrives in the upper canopy of tropical evergreen rainforests and monsoon forests within a wide range of elevations, from 330 to 6,000 ft (100–1,850 m). They are endemic to the Western Ghats, a substantial mountain range in India that stretches 990 mi (1,600 km) and is interrupted only by the 25 mi (40 km) wide Palghat gap. The Western Ghats traverse the states of Karnataka, Tamil Nadu, and Kerala along the southwestern coast of the Indian peninsula. Lion-tailed macaques are diurnal and live in groups of 10 to 20 individuals.

Statement 2 is correct: The Malabar Civet is indeed one of the world’s rarest mammals. It is a nocturnal and elusive animal endemic to the Western Ghats of India. The civetone extract from its scent gland is used in medicine and perfumes.

Conservation Status:

  • IUCN Red List: Critically Endangered.
  • Wildlife (Protection) Act, 1972: Schedule I.
  • CITES: Appendix III.

Statement 3 is correct: Scientists from the Zoological Survey of India (ZSI) have reported the first photographic evidence of the Sambar (Rusa unicolor) in the Pangolakha Wildlife Sanctuary (PWLS) and adjoining areas of East Sikkim. Sambar deer have several subspecies that vary in size and appearance. They are known as the largest Oriental deer, with some adult males reaching weights of up to 550 kg. The males can be identified by their antlers and are slightly larger and darker than females and young individuals. Sambar deer are elusive and most active at dusk and night. They are preyed upon by Indian leopards, Bengal tigers, and dholes, making them an important component of the ecosystem.

IUCN Red List status: Vulnerable.

Source – https://journalsofindia.com/malabar-civet/ https://neprimateconservancy.org/lion-tailed-macaque/

15. Which of the following organisms perform waggle dance for others of their kin to indicate the direction and the distance to a source of their food?

(a) Butterflies
(b) Dragonflies
(c) Honeybees
(d) Wasps

Answer – The correct answer is C, i.e Honeybees.

Honey bees utilize a sophisticated form of spatial referential communication. Through their “waggle dance,” they convey the direction, distance, and quality of a resource to other members of the hive. This dance encodes celestial cues, retinal optic flow, and relative food value into motion and sound, enabling effective communication within the nest.

Source – https://pib.gov.in/PressReleasePage.aspx?PRID=1879603

16. Consider the following statements:

  1. Some mushrooms have medicinal properties.
  2. Some mushrooms have psycho active properties.
  3. Some mushrooms have insecticidal properties.
  4. Some mushrooms have bioluminescent properties.

How many of the above statements are correct?

(a) Only one
(b) Only two
(c) Only three
(d) All four

Answer – The correct answer is D, i.e All four.

The North Eastern Region Community Resource Management Society (NERCRMS), operating under the Ministry of DONER, aims to empower Nari Shakti through skill development. Their efforts have enabled women in rural areas of the North Eastern regions to earn a livelihood through mushroom cultivation.

Statement 1 is correct: Mushrooms possess antibacterial properties, enhance the immune system, and aid in lowering cholesterol. Moreover, they are valuable sources of bioactive compounds. These properties make certain mushroom extracts beneficial for promoting human health and are used as dietary supplements.

Statement 2 is correct: The effects of psilocybin, a compound found in certain mushrooms, are generally similar to those of LSD. They include altered perception of time and space, as well as intense changes in mood and feeling.

Statement 3 is correct: Endophytic fungi have the ability to colonize internal plant tissues and can act as control agents in processes such as induced resistance and alleviating abiotic stresses. They may function as biological agents or elicitors in these capacities.

Statement 4 is correct: A mushroom documentation project conducted in the forests of Northeast India uncovered not only 600 varieties of fungi but also led to the discovery of a new species: a bioluminescent mushroom. The newly identified species, named Roridomyces phyllostachydis, was first observed on a wet August night near a stream in Meghalaya’s Mawlynnong and later in Krang Shuri, located in West Jaintia Hills district. It adds to the existing list of 97 known species of bioluminescent fungi worldwide.

Source – https://www.ncbi.nlm.nih.gov/pmc/articles/PMC4320875/#:~:text=Mushrooms%20act%20as%20antibacterial%2C%20immune,are%20found%20as%20dietary%20supplements

17. Consider the following statements regarding the Indian squirrels :

  1. They build nests by making burrows in the ground.
  2. They store their food materials like nuts and seeds in the ground.
  3. They are omnivorous.

How many of the above statements are correct?

(a) Only one
(b) Only two
(c) All three
(d) None

Answer – The correct answer is B, i.e Only two.

Scientists have discovered fossils of a newly identified genus and species of small mammal resembling squirrels, known as treeshrews, in Ramnagar, Jammu and Kashmir.

Statement 1 is not correct: Treeshrews are typically solitary animals, although they may come together in pairs during the breeding season. They construct multiple nests, called dreys, within a single breeding season. These nests are made of leaves and twigs and are typically built in tall, extensively branched trees within the higher canopy.

Statement 2 is correct: Nuts serve as their primary food source throughout the year, especially when they are preparing for winter. Squirrels gather nuts during the late summer and fall, and they store them by burying them underground. This allows them to retrieve the stored nuts during winter when food is scarce.

Statement 3 is correct: Treeshrews primarily feed on eggs, fruits, and insects as part of their diet.

Source – https://pib.gov.in/PressReleaseIframePage.aspx?PRID=1838021

18. Consider the following statements

  1. Some microorganisms can grow in environments with temperature above the boiling point of water.
  2. Some microorganisms can grow in environments with temperature below the freezing point of water.
  3. Some microorganisms can grow in highly acidic environment with a pH below 3.

How many of the above statements are correct?

(a) Only one
(b) Only two
(c) All three
(d) None

Answer – The correct answer is C, i.e All three.

Statement 1 is correct: Archaea, one of the three domains of life, encompasses a highly diverse and abundant group of prokaryotes. It includes numerous “extremophiles” that thrive in extreme environments such as hot springs, salt lakes, and submarine volcanic habitats.

Statement 2 is correct: Microorganisms inhabit every part of the biosphere, and some are capable of thriving at low temperatures, even below freezing. These microorganisms can be found in the sea, high mountains, and unfortunately, even in refrigerators, where they can spoil food or contaminate it as pathogens.

Psychrophiles, which include bacteria, lichens, snow algae, phytoplankton, fungi, and insects, are adapted to cold environments. Bacterial examples of cold-tolerant species include Arthrobacter sp., Psychrobacter sp., and members of the genera Halomonas, Pseudomonas, Hyphomonas, and Sphingomonas. Psychrophilic microorganisms have successfully colonized permanently cold environments ranging from the deep sea to mountain and polar regions. Some of these organisms, depending on their optimal growth temperature, are also referred to as psychrotolerant or psychrotroph.

Statement 3 is correct: Acidophiles are organisms that thrive in acidic environments and include certain types of eukaryotes, bacteria, and archaea. They can be found in various acidic habitats, such as sulfuric pools, geysers, areas contaminated by acid mine drainage, and even our own stomachs.

Source – https://www.wwf.org.uk/learn/fascinating-facts/orangutans

19. Which one of the following makes a tool with a stick to scrape insects from a hole in a tree or a log of wood?

(a) Fishing cat
(b) Orangutan
(c) Otter
(d) Sloth bear

Answer – The correct answer is Orangutan.

The term orangutan, originating from the Malay language, translates to “person of the forest.” These primates, characterized by their long hair and orangish coloration, are highly intelligent and share close evolutionary ties with humans. They are exclusively found in the regions of Sumatra and Borneo. Notably, certain Sumatran orangutans exhibit tool usage, employing sticks to extract termites, ants, or bees from tree holes. These resourceful creatures have also been observed crafting leaf “gloves” to handle prickly fruits or thorny branches.

Source – https://www.wwf.org.uk/learn/fascinating-facts/orangutans

20. Consider the following:

  1. Aerosols
  2. Foam agents
  3. Fire retardants
  4. Lubricants

In the making of how many of the above are hydrofluorocarbons used?

(a) Only one
(b) Only two
(c) Only three
(d) All four

Answer – The correct answer is C, i.e Only three.

Hydrofluorocarbons (HFCs) are a group of synthetic chemicals primarily utilized for cooling and refrigeration purposes. These chemicals were developed as substitutes for ozone-depleting substances present in the atmosphere, which are currently being phased out under the Montreal Protocol on Substances that Deplete the Ozone Layer.

HFCs are entirely human-made and not naturally occurring. HFCs are synthetic gases used in air conditioning systems, aerosol propellants, foam blowing agents, solvents, and flame retardants. Most HFCs are enclosed within equipment, and their emissions result from factors such as wear and tear, faulty maintenance, or leakage at the end of a product’s lifespan. It’s important to note that chlorofluorocarbons (CFCs) are also found in refrigerant lubricants.

Polyolester oil, which are wax free oils are suggested for use with chlorine free HFC systems as they provide better lubrication and stability and are more miscible with HFC refrigerants. HFCs are not used for lubrication. Hence, 4 is incorrect.

Source – https://en.wikipedia.org/wiki/Polyolester

21. Consider the following statements:

Statement-1:
Interest income from the deposits in Infrastructure Investment Trusts (InvITs) distributed to their investors is exempted from tax, but the dividend is taxable.

Statement-II :
InvITs are recognized as borrowers under the ‘Securitization and Recon- struction of Financial Assets and Enforcement of Security Interest Act, 2002’.

Which one of the following is correct in respect of the above statements?

(a) Both Statement-I and Statement-II are correct and Statement-II is the correct explanation for Statement-I
(b) Both Statement-I and Statement-II are correct and Statement-II is not the correct explanation for Statement-I
(c) Statement-I is correct but Statement-II is incorrect
(d) Statement-I is incorrect but Statement-II is correct

Answer – The correct answer is D.

Statement 1: This statement is not correct.

  • Both interest and dividends are subject to taxation based on your income tax slab. This applies when the Infrastructure Investment Trust (InvIT) has chosen to be taxed under section 115BAA of the Act.
  • In some cases, a portion of your investment in an InvIT is utilized as a loan, and the InvIT pays you interest on it.

Statement 2: This statement is correct.

  • Amendments have been made to the SARFAESI Act (Securitization and Reconstruction of Financial Assets and Enforcement of Security Interest Act) and the Recovery of Debts Act to provide additional enforcement options for domestic lenders.
  • These amendments expand the definition of ‘borrower’ under these acts to include a pooled investment vehicle.
  • When combined with the definition of ‘secured creditor’ under the SARFAESI Act, this enables a debenture trustee overseeing listed secured debt securities issued by an InvIT or REIT (Real Estate Investment Trust) to access the protections and utilize the enforcement mechanisms provided.
  • Similarly, eligible lenders now have the benefit of the Recovery of Debts Act in relation to debt securities issued by an InvIT or REIT as well.

22. Consider the following statements :

Statement-1 :
In the post-pandemic recent past, many Central Banks worldwide had carried out interest rate hikes.

Statement-II:
Central Banks generally assume that they have the ability to counteract the rising consumer prices via monetary policy means.

Which one of the following is correct in respect of the above statements?

(a) Both Statement-I and Statement-II are correct and Statement-II is the correct explanation for Statement-I
(b) Both Statement-I and Statement-II are correct and Statement-II is not the correct explanation for Statement-I
(c) Statement-I is correct but Statement-II is incorrect
(d) Statement-I is incorrect but Statement-II is correct

Answer – The correct answer is A.

Statement 1: This statement is correct.

In the past, global economic shocks were severe but spaced out over time. However, this pattern changed in the third decade of the current millennium, as the global economy has experienced at least three shocks since 2020.

The sequence began with the contraction of global output caused by the pandemic, followed by the Russian-Ukraine conflict, which resulted in a worldwide surge in inflation. In response, central banks worldwide, led by the Federal Reserve, implemented synchronized policy rate hikes to control inflation. The rate hike by the US Fed attracted capital into the US markets, leading to the appreciation of the US Dollar against most currencies. As a consequence, Current Account Deficits (CAD) widened, and net importing economies faced increased inflationary pressures.

Statement 2: This statement is correct.

The central bank has the authority to provide funds to the banking system and charge interest on those funds. Due to its monopoly power over money issuance, the central bank has full control over determining the interest rate.
Source – https://www.lexology.com/commentary/banking-financial-services/india/azb-partners/fpi-investment-in-debt-instruments-by-reits-and-invits

23. Consider the following statements:

Statement-1:
Carbon markets are likely to be one of the most widespread tools in the fight against climate change.

Statement-Il
Carbon markets transfer resources from the private sector to the State.

Which one of the following is correct in respect of the above statements?

(a) Both Statement-I and Statement-II are correct and Statement-II is the correct explanation for Statement-1
(b) Both Statement-I and Statement-11 are correct and Statement-11 is not the correct explanation for Statement-I
(c) Statement-I is correct but Statement-II is incorrect
(d) Statement-I is incorrect but Statement-II is correct

Answer – The correct answer is B.

Statement 1: This statement is correct.

  • Carbon markets have finally become one of the most widely adopted tools in the battle against climate change. By the end of 2021, over 21% of global emissions were encompassed by some form of carbon pricing, an increase from 15% in 2020. More and more businesses are now required to pay regulatory fees for their carbon dioxide emissions, and investors are also showing increased interest, leading to a 164% growth in market trading last year, reaching €760 billion ($897 billion).

Statement 2: This statement is correct.

  • Similar to taxes, carbon markets facilitate the transfer of resources from the private sector to the state, which can be disconcerting for proponents of limited government involvement. Furthermore, higher carbon prices have the potential to drive up consumer prices, which can be a source of frustration for voters. Additionally, these prices can impact the profit margins of companies that contribute to political parties.

However, just because the carbon markets are facilitating transfer of resources from the private to the state, it didn’t result in carbon market being used as a tool to fight climate change. There are many other factors.

Several factors have contributed to the widespread adoption of carbon markets as a tool in combating climate change. Some key factors include:

  • International Climate Agreements
  • Recognition of Economic Incentives
  • Business Engagement
  • Growing Investor Interest
  • Public Awareness
  • Technological Advancements

Source – https://www.economist.com/finance-and-economics/2022/05/26/carbon-markets-are-going-global

24. Which one of the following activities of the Reserve Bank of India is considered to be part of ‘sterilization?

(a) Conducting ‘Open Market Operations’
(b) Oversight of settlement and payment systems
(c) Debt and cash management for the Central and State Governments
(d) Regulating the functions of Non-banking Financial Institutions

Answer – The correct answer is A, i.e Conducting “Open Market Operations”

Open market operations refer to the buying and selling of government securities (such as Treasury bills and bonds) by the central bank in the open market.

  • When the central bank wants to inject liquidity into the economy, it purchases government securities from commercial banks and other financial institutions, thereby increasing the money supply.
  • Conversely, when the central bank wants to reduce liquidity, it sells government securities, effectively withdrawing money from circulation.

Sterilization refers to the offsetting actions taken by the central bank to counterbalance the impact of its foreign exchange interventions on the domestic money supply

Sterilization can involve conducting operations such as selling or purchasing government securities in the open market, but it encompasses a broader concept.

In the context of sterilization, the central bank may engage in open market operations to absorb or release liquidity from the market, aiming to offset the impact of its foreign exchange interventions. These operations help maintain monetary stability by managing the money supply and controlling inflationary pressures.
Therefore, open market operations can be utilized as a means of implementing sterilization, but they are not inherently the same thing. Open market operations encompass a broader set of actions related to the buying and selling of government securities, while sterilization specifically refers to actions taken to offset the effects of foreign exchange interventions on the domestic money supply.

25. Consider the following markets :

  1. Government Bond Market
  2. Call Money Market
  3. Treasury Bill Market
  4. Stock Market

How many of the above are included/ in capital markets?

(a) Only one
(b) Only two
(c) Only three
(d) All four

Answer – The correct answer is B, i.e Only two.

The money market and the capital market are distinct segments within the financial market, catering to different time horizons of funding requirements.

The money market primarily serves short-term financial needs, typically up to 364 days, while the capital market addresses long-term financing needs exceeding 365 days.

Money market instruments consist of Bills of Exchange or Commercial Bills, Treasury Bills (T-Bills), Commercial Papers (CP), Certificates of Deposits (CD), Repurchase Agreements, Banker’s Acceptance, and Call & Notice Money. These instruments facilitate short-term borrowing, lending, and liquidity management in the financial market.
On the other hand, capital market instruments include Bonds and Stocks, which are utilized for long-term financing and investment purposes, representing ownership or debt obligations in corporations or governments.

26. Which one of the following best describes the concept of ‘Small Farmer Large Field’?

(a) Resettlement of a large number of people, uprooted from their countries due to war, by giving them large cultivable land which they cultivate collectively and share the produce
(b) Many marginal farmers in an area organize themselves into groups and synchronize and harmonize selected agricultural operations
(c) Many marginal farmers in an area together make a contract with a corporate body and surrender their land to the corporate body for a fixed term for which the corporate body makes a payment of agreed amount to the farmers
(d) A company extends loans, technical knowledge and material inputs to a number of small farmers in an area so that they produce the agricultural commodity required by the company manufacturing for its process and commercial production

Answer – The correct answer is B.

The Small Farmers Large Field (SFLF) model is a synchronized collective action approach aimed at improving the livelihoods of small farmers in India.

This model addresses the challenges faced by small and marginal farmers, such as the lack of bargaining power in the supply chain and diseconomies of scale. By organizing themselves into groups and synchronizing their operations, small farmers can benefit from economies of scale and overcome these disadvantages.

27. Consider the following statements :

  1. The Government of India provides Minimum Support Price for niger (Guizotia abyssinica) seeds.
  2. Niger is cultivated as a Kharif crop.
  3. Some tribal people in India use niger seed oil for cooking.

How many of the above statements are correct?

(a) Only one
(b) Only two
(c) All three
(d) None

Answer – The correct answer is C, i.e All three.

Among the 14 kharif crops that receive a minimum support price (MSP) from the government, one peculiar plant stands out—niger or ramtil (Guizotia abyssinica). This lesser-known oilseed garners one of the highest MSPs, determined by considering production costs and market demand. Hence, Statement 1 and 2 are correct.

The tribal communities utilize niger seed oil for cooking purposes and use the residue, known as press cake, as livestock feed. Additionally, they consume the seeds as a condiment. Niger seed oil’s commercial demand is attributed to its medicinal properties, attracting industries such as cosmetics, perfumeries, and related sectors. Hence, Statement 3 is also correct.

Source – https://www.downtoearth.org.in/news/agriculture/illusive-oilseed-india-s-niger-seed-cultivation-is-declining-here-is-why-84380

28. Consider the investments in the following assets:

  1. Brand recognition
  2. Inventory
  3. Intellectual property
  4. Mailing list of clients

How many of the above are considered intangible investments?

(a) Only one
(b) Only two
(c) Only three
(d) All four

Answer – The correct answer is C, i.e Only three.

Intangible assets can be classified as either identifiable or unidentifiable, as well as definite or indefinite. Identifiable assets are those that can be separated from the company and retain their existence, whereas unidentifiable assets cannot be separated. Definite intangible assets have a specific lifespan, while indefinite intangible assets do not have a predetermined duration.

Several examples of intangible assets encompass brand recognition, goodwill, and intellectual property. Intellectual property includes patents, domain names, confidential information, inventions, names, and other similar assets. These intangible assets hold significant value for organizations in various industries. Hence, 1, 3 and 4 are correct.

A tangible asset is an asset characterized by its physical substance. It encompasses a wide range of items, such as inventory, buildings, rolling stock, manufacturing equipment or machinery, and office furniture. Tangible assets can be broadly categorized into two types: inventory and fixed assets. Hence, 2 is NOT correct.

Inventory refers to the goods or materials held by a company for production, sale, or use in its operations. These can include raw materials, work-in-progress items, or finished goods that are ready for distribution or sale.
Fixed assets, on the other hand, are long-term assets that are used in the production or operation of a business. They are not intended for sale in the ordinary course of business and provide ongoing benefits over an extended period. Examples of fixed assets include buildings, machinery, vehicles, and furniture that are utilized in the company’s day-to-day operations.

29. Consider the following:

  1. Demographic performance
  2. Forest and ecology
  3. Governance reforms
  4. Stable government
  5. Tax and fiscal efforts

For the horizontal tax devolution, the Fifteenth Finance Commission used how many of the above as criteria other than population area and income distance?

(a) Only two
(b) Only three
(c) Only four
(d) All five

Answer – The correct answer is B, i.e Only three.

The criteria for the distribution of central taxes among states for the 2021-26 period remain the same as those for the 2020-21 period.

  • Income distance: The income distance criterion measures the difference between a state’s income and the income of the state with the highest income. The income of each state is calculated as the average per capita Gross State Domestic Product (GSDP) during the three-year period from 2016-17 to 2018-19. States with lower per capita income receive a higher share to ensure equity among states.
  • Demographic performance: The Commission used 2011 population data, as mandated by its Terms of Reference, to assess demographic performance. This criterion rewards states that have made efforts to control their population, with states having a lower fertility ratio receiving higher scores. Hence, 1 is correct.
  • Forest and ecology: The criterion for forest and ecology takes into account the proportion of dense forest in each state relative to the total dense forest area of all states. This criterion recognizes the contribution of states in preserving and maintaining dense forests. Hence, 2 is correct.
  • Tax and fiscal efforts: The tax and fiscal efforts criterion aims to reward states that exhibit higher efficiency in tax collection. It is calculated as the ratio of average per capita own tax revenue to the average per capita state GDP during the three-year period from 2016-17 to 2018-19. Hence, 5 is correct.
  • Statements 3 and 4 are not correct: Stable government and government reforms are not considered as criteria for tax devolution.

Source – https://prsindia.org/policy/report-summaries/report-15th-finance-commission-2021-26

30. Consider the following infrastructure sectors:

  1. Affordable housing
  2. Mass rapid transport
  3. Health care
  4. Renewable energy

On how many of the above does UNOPS Sustainable Investments in Infrastructure and Innovation (S3) initiative focus for its investments?

(a) Only one
(b) Only two
(c) Only three
(d) All four

Answer – The correct answer is c, i.e Only three.

Affordable Housing, Health Care, and Renewable Energy are the focus areas of the UNOPS Sustainable Investments in Infrastructure and Innovation initiative. These investments aim to promote sustainable development and address key challenges in these sectors.

2 is not correct: Mass Rapid Transport is not included as part of this initiative.

Source – Link

31. In essence, what does ‘Due Process of Law’ mean?

(a) The principle of natural justice.
(b) The procedure established by law
(c) Fair application of law
(d) Equality before law

Answer – The correct answer is C, i.e Fair application of Law.

Due process refers to the essential principle that legal proceedings must adhere to established rules and principles, ensuring fair treatment for individuals involved. It applies to both civil and criminal cases.

In jurisdictions with well-developed legal systems, individuals have the rightful expectation that the rights guaranteed by their constitutions will be upheld and applied fairly. The concept of due process reflects the desired relationship between individuals and their local, state, and federal governments, emphasizing the protection of individual rights from unjust infringement.

32. Consider the following statements:

Statement-1:
In India, prisons are managed by State Governments with their own rules and regulations for the day-to-day administration of prisons.

Statement-II:
In India, prisons are governed by the Prisons Act, 1894 which expressly kept the subject of prisons in the control of Provincial Governments.

Which one of the following is correct in respect of the above statements?

(a) Both Statement-1 and Statement-11 are correct and Statement-II is the correct explanation for Statement-l
(b) Both Statement-I and Statement-II are correct and Statement-11 is not the correct explanation for Statement-I
(c) Statement-I is correct but Statement-II is incorrect
(d) Statement-I is incorrect but Statement-II is correct

Answer – The correct answer is A.

The subject of ‘Prisons’ is listed under the State List of the Seventh Schedule to the Constitution of India.

  • It falls within the jurisdiction of the State Governments, who have exclusive authority over the management and administration of prisons.
  • The State Governments govern prisons through the Prisons Act, 1894, and the Prison Manuals specific to each state. As a result, the responsibility and authority to amend existing prison laws, rules, and regulations lie primarily with the State Governments.

Hence, both the statements are correct and Statement II is the correct explanation of Statement I.

33. Which one of the following statements best reflects the Chief purpose of the ‘Constitution’ of a country?

(a) It determines the objective for the making of necessary laws.
(b) It enables the creation of political offices and a government.
(c) It defines and limits the powers of government.
(d) It secures social justice, social equality and social security

Answer – The correct answer is A.

  • The chief function of a constitution is to establish a foundational framework consisting of essential rules that facilitate coordination among the members of a society.
  • The second important function of a constitution is to determine the allocation of decision-making power within a society. It outlines the structure and composition of the government and establishes the mechanisms through which power is exercised and decisions are made.
  • The third essential function of a constitution is to establish boundaries that restrict the extent to which a government can impose its authority on its citizens. These limits are of utmost importance as they define the fundamental rights and freedoms that individuals possess, ensuring that the government never oversteps these boundaries.
  • The fourth crucial function of a constitution is to empower the government to address the aspirations of a society and establish the necessary conditions for a just and equitable society. By providing a framework for governance, the constitution enables the government to work towards the betterment of society, promote social justice, and create an environment where all individuals can thrive and fulfill their potential.

Source – https://ncert.nic.in/textbook/pdf/keps201.pdf

34. In India, which one of the following Constitutional Amendments was widely believed to be enacted to overcome the judicial interpretations of the Fundamental Rights?

(a) 1st Amendment.
(b) 42nd Amendment
(c) 44th Amendment
(d) 86th Amendment

Answer – The correct answer is A, i.e 1st Amendment.

The validity of the First Amendment Act to the Constitution was challenged on the ground that it purported to abridge the fundamental rights under Part 3 of the Constitution of India.

35. Consider the following organizations/ bodies in India :

  1. The National Commission for Backward Classes
  2. The National Human Rights Commission
  3. The National Law Commission.
  4. The National Consumer Disputes Redressal Commission

How many of the above are constitutional bodies?

(a) Only one
(b) Only two
(c) Only three
(d) All four

Answer – The correct answer is A, i.e Only one.

1 is correct: The 102nd Constitution Amendment Act, 2018 provides constitutional status to the National Commission for Backward Classes (NCBC)Article 338B. It empowers the NCBC to examine complaints and welfare measures related to socially and educationally backward classes. Prior to this amendment, the NCBC was a statutory body operating under the Ministry of Social Justice and Empowerment.

2 is not correct: The NHRC (National Human Rights Commission) is not a statutory body. It was established on October 12, 1993, under the Protection of Human Rights Act (PHRA), 1993. The PHRA serves as the legal framework for the functioning of the NHRC. This act has been amended in 2006.

3 is not correct: The Law Commission of India is a non-statutory body formed through a government notification by the Ministry of Law & Justice, Department of Legal Affairs. It has specific terms of reference to conduct research in the field of law and make recommendations to the government.
4 is not correct: The National Consumer Disputes Redressal Commission (NCDRC) is a quasi-judicial commission established under the provisions of the Consumer Protection Act, 1986. It was established in 1988 and has its headquarters in New Delhi. The NCDRC is headed by a retired or sitting judge of the Supreme Court.

36. Consider the following statements:

  1. If the election of the President of India is declared void by the Supreme Court of India, all acts done by him/her in the performance of duties of his/her office of President before the date of decision become invalid.
  2. Election for the post of the President of India can be postponed on the ground that some Legislative Assemblies have been dissolved and elections are yet to take place.
  3. When a Bill is presented to the President of India, the Constitution prescribes time limits within which he/she has to declare his/her assent.

How many of the above statements are correct?

(a) Only one
(b) Only two
(c) All three
(d) None

Answer – The correct answer is D, i.e None.

Statement 1 is NOT correct.

Article 71 (2) in the Constitution Of India, if the election of a person as President or Vice President is declared void by the Supreme court, acts done by him in the exercise and performance of the powers and duties of the office of President or Vice President, as the case may be, on or before the date of the decision of the Supreme Court shall not be invalidated by reason of that declaration.

Statement 2 is absurd.

Statement 3 is incorrect.

According to Article 111 of the Constitution, once a Bill has been passed by both Houses of Parliament, it is presented to the President for assent. The President has the authority to either give assent to the Bill or withhold assent from it. However, if the Bill is not a Money Bill, the President has the option to return it to the Houses with a message requesting them to reconsider specific provisions or amendments recommended in the message. In such cases, the Houses will reconsider the Bill accordingly, and if it is passed again with or without amendments and presented to the President, the President cannot withhold assent.
It should be noted that the Constitution does not specify a specific time limit within which the President must declare his or her assent. Therefore, statement 3 is not correct.

37. With reference to Finance Bill and Money Bill in the Indian Parliament, consider the following statements:

  1. When the Lok Sabha transmits Finance Bill to the Rajya Sabha, it can amend or reject the Bill.
  2. When the Lok Sabha transmits Money Bill to the Rajya Sabha, it cannot amend or reject the Bill, it can only make recommendations.
  3. In the case of disagreement between the Lok Sabha and the Rajya Sabha, there is no joint sitting for Money Bill, but a joint sitting becomes necessary for Finance Bill.

How many of the above statements are correct?

(a) Only one
(b) Only two
(c) All three
(d) None

Answer – The correct answer is A, i.e Only one.

A Finance Bill is classified as a Money Bill according to Article 110 of the Constitution. In contrast, a Financial Bill is considered an ordinary bill because it addresses not only monetary matters but also non-monetary matters.

Statement 1 is NOT correct: The Finance Bill can be introduced only in Lok Sabha. However, the Rajya Sabha can recommend amendments in the Bill. The bill has to be passed by the Parliament within 75 days of its introduction.

Statement 2 is correct: Rajya Sabha can only give recommendations to the Money Bill, which can either be accepted or rejected by Lok Sabha. The Rajya Sabha has to do so within 14 days, which is an otherwise case, is deemed to have been passed by both the houses of the Parliament.
Statement 3 is NOT correct: Joint sitting is not NECESSARY in case of a disagreement between Lok Sabha and Rajya Sabha, although there is a provision for a joint-sitting by the President for Financial Bill Type II, which doesn’t include matters as contained in Article 110.

38. Consider the following statements:

Once the Central Government notifies an area as a ‘Community Reserve’

  1. the Chief Wildlife Warden of the State becomes the governing authority of such forest hunting is not allowed in such area
  2. people of such area are allowed to collect non-timber forest produce
  3. people of such area are allowed traditional agricultural practices

How many of the above statements are correct?

(a) Only one
(b) Only two
(c) Only three
(d) All four

Answer – The correct answer is B, i.e Only two.

Statement 1 is correct: When the central government designates an area as a community reserve, as per Section 33 of the Wildlife Protection Act (WLPA) Act, the Chief Wildlife Warden of the respective state becomes the governing authority of the forest. The consent of the Chief Wildlife Warden is required for all decisions related to the area.

Statement 2 is correct, but Statement 3 and 4 are not correct: Once a forest is declared as a community reserve, hunting is prohibited in that area. Similarly, the collection of non-timber forest produce and agricultural practices like jhum cultivation are also restricted or disallowed in community reserves.

39. With reference to ‘Scheduled Areas’ in India, consider the following statements :

  1. Within a State, the notification of an area as Scheduled Area takes place through an Order of the President.
  2. The largest administrative unit forming the Scheduled Area is the District and the lowest is the cluster of villages in the Block.
  3. The Chief Ministers of the concerned States are required to submit annual reports to the Union Home Ministry on the administration of Scheduled Areas in the States.

How many of the above statements are correct?

(a) Only one
(b) Only two
(c) All three
(d) None

Answer – The correct answer is B, i.e Only two.

Statement 1 is correct: The designation of “Scheduled Areas” in a State is done through a notified order of the President, after consultation with the concerned State Government.

Statement 2 is correct: The district is the largest administrative unit within the scheduled areas, and the block consists of clusters of villages, forming the lowest administrative unit.

Statement 3 is not correct: Instead, the Governor of each State with Scheduled Areas is required to submit an annual report or as requested by the President regarding the administration of the Scheduled Areas in that State. The executive power of the Union can be exercised to provide directions to the State regarding the administration of these areas.

40. Consider the following statements:

Statement-I :
The Supreme Court of India has held in some judgments that the reservation policies made under Article 16(4) of the Constitution of India would be limited by Article 335 for maintenance of efficiency of administration.

Statement-II :
Article 335 of the Constitution of India defines the term ‘efficiency of administration’.

Which one of the following is correct in respect of the above statements?

(a) Both Statement-I and Statement-II are correct and Statement-II is the correct explanation for Statement-I
(b) Both Statement-I and Statement-II are correct and Statement-II is not the correct explanation for Statement-I
(c) Statement-1 is correct but Statement-II is incorrect
(d) Statement-1 is incorrect but Statement-II is correct

Answer – The correct answer is C.

Statement 1 is correct:

The Supreme Court of India has indeed held in some judgments that the reservation policies made under Article 16(4) of the Constitution of India may be limited by Article 335 for the maintenance of efficiency in administration. The judgements include:

  • Indra Sawhney and Others v Union of India and Others, 1993
  • M Nagaraj and Others v Union of India and Others, 2006

Article 16(4) allows for reservations in public employment for socially and educationally backward classes of citizens. However, Article 335 emphasizes that while implementing these reservations, the claims of Scheduled Castes (SCs) and Scheduled Tribes (STs) should be taken into consideration, keeping in mind the maintenance of efficiency in the administration.

Statement 2 is incorrect:

Article 335 of the Constitution of India does not provide a specific definition for the term ‘efficiency of administration’. Instead, it specifies that the claims of the members of the Scheduled Castes (SCs) and the Scheduled Tribes (STs) should be taken into consideration while ensuring the efficiency of administration.

41. In which one of the following regions was Dhanyakataka, which flourished as a prominent Buddhist centre under the Mahasanghikas, located?

(a) Andhra
(b) Gandhara
(c) Kalinga
(d) Magadha

Answer – The correct answer is A, i.e Andhra.

Amaravati, located in Andhra Pradesh, holds historical significance as the ancient town of Dhanyakataka. It served as a prominent center in the Deccan region and was the capital of the later Satavahanas, as attested by numerous inscriptions. This region also housed a significant Buddhist settlement, adding to its cultural and religious importance. Spanning from the 2nd century BCE to the 2nd/3rd century CE, Amaravati witnessed six distinct occupational periods, showcasing its enduring historical legacy.

42. With reference to ancient India, consider the following statements :

  1. The concept of Stupa is Buddhist in origin.
  2. Stupa was generally a repository of relics.
  3. Stupa was a votive and commemorative structure in Buddhist tradition.

How many of the statements given above are correct?

(a) Only one
(b) Only two
(c) All three
(d) None

Answer – The correct answer is C, i.e All three.

Statement 1 is correct:

Stupas are indeed closely associated with Buddhism and hold a significant place in Buddhist architecture and religious rituals. Initially constructed to enshrine the relics of the Buddha, stupas evolved into important pilgrimage sites for Buddhists. They symbolize the enlightened mind of the Buddha and serve as objects of veneration and meditation. While variations of stupas can be found in other cultures and religions, the concept of the stupa as commonly understood today originates from the Buddhist tradition.

Statement 2 is correct:

Stupas were commonly built as structures to house and safeguard sacred relics. These relics could include bodily remains of the Buddha, such as bone fragments or ashes, as well as objects associated with the Buddha or other revered figures in Buddhism. The relics were often enshrined within the stupa, typically in a central chamber or multiple chambers. The presence of relics within the stupa made it a significant pilgrimage site and an object of veneration for Buddhists. Therefore, a stupa generally served as a repository of relics.

Statement 3 is correct:

Stupas served various purposes within the Buddhist tradition, including being votive and commemorative structures. Votive refers to offerings or dedications made by devotees to express their devotion, gratitude, or seek blessings. Buddhists would frequently visit stupas to make offerings such as flowers, incense, or lamps as acts of reverence and devotion. Stupas also functioned as commemorative structures, representing significant events or important figures in Buddhist history. For instance, some stupas were constructed to commemorate the birth, enlightenment, or parinirvana (passing away) of the Buddha. They were built as symbols of remembrance and to honor the teachings and legacy of the Buddha or other revered figures in Buddhism. Therefore, statement 3 is correct.

43. With reference to ancient South India, Korkai, Poompuhar and Muchiri were well known as

(a) capital cities
(b) ports
(c) centres of iron-and-steel trailing
(d) shrines of Jain Tirthankaras

Answer – The correct answer is B, i.e Ports.

  • Korkai, situated in present-day Tamil Nadu, served as an ancient port city and a notable hub of trade and commerce in the region. It played a crucial role in maritime activities and facilitated commercial exchanges.
  • Poompuhar, also known as Kaveripattinam or Kaveripoompattinam, was another significant port city located along the banks of the River Kaveri. It was renowned as a prominent center for maritime trade and finds mention in ancient Tamil literature.
  • Muchiri, also referred to as Musiri, was a coastal town that held strategic importance as an ancient port. While these locations might have had additional features such as being centers for iron-and-steel production or hosting shrines of Jain Tirthankaras, their primary identification remains as significant port cities.

44. Which one of the following explains the practice of ‘Vattakirutal’ as mentioned in Sangam poems?

(a) Kings employing women bodyguards
(b) Learned persons assembling in royal courts to discuss religious and philosophical matters
(c) Young girls keeping watch over agricultural fields and driving away birds and animals
(d) A king defeated in a battle committing ritual suicide by starving himself to death

Answer – The correct answer is D.

Vatakkiruttal, also known as Vadakiruthal or vadakiruttal, was a ritualistic practice in ancient Tamil culture that involved fasting until death. This practice was particularly prevalent during the Sangam age, an era in Tamil history known for its literary works and cultural advancements. The Tamil kings, driven by the desire to uphold their honor and prestige, were willing to face death while fasting in a northward direction (‘Vatakkiruttal’). They would not retreat or turn their back in battle, demonstrating their unwavering determination and courage in the face of adversity.

45. Consider the following dynasties

  1. Hoysala
  2. Gahadavala
  3. Kakatiya
  4. Yadava

How many of the above dynasties established their kingdoms in early eighth century AD?

(a) Only one
(b) Only two
(c) Only three
(d) None

Answer – The correct answer is D, i.e None.

DynastyTime-Period
Hoysala11th Century AD
Gahadavala10th-11th Century AD
Kakatiya12th Century AD
Yadava12th-13th Century AD

46. With reference to ancient Indian History, consider the following pairs :

Literary – work Author

  1. Devichandragupta : Bilhana
  2. Hammira-Mahakavya : Nayachandra Suri
  3. Milinda-panha : Nagarjuna
  4. Nitivakyamrita : Somadeva Sufi

How many of the above pairs are correctly matched?

  • (a) Only one
  • (b) Only two
  • (c) Only three
  • (d) All four

Answer – The correct answer is B, i.e Only two.

Literary WorkAuthor
DevichandraguptaBilhana Vishakhadatta (Incorrectly Matched)
Hammira-MahakavyaNayachandra Suri (Correctly Matched)
Milinda-panha Nagarjuna – Nagarsena (Incorrectly Matched)
NitivakyamritaSomadeva Suri (Correctly Matched)

47. “Souls are not only the property of animal and plant life, but also of rocks, running water and many other natural objects not looked on as living by other religious sects.”

The above statement reflects one of the core beliefs of which one of the following religious sects of ancient India?

(a) Buddhism
(b) Jainism
(c) Shaivism
(d) Vaishnavism

Answer – The correct answer is B, i.e Jainism.

The Jain religion upholds the belief in reincarnation, where the cycle of birth, death, and rebirth is influenced by an individual’s karma. According to Jains, negative karma arises from causing harm to living beings. To steer clear of accumulating bad karma, Jains diligently adhere to ahimsa, a rigorous principle of nonviolence. Jains recognize the presence of souls not only in humans but also in plants, animals, and certain inanimate entities like air and water. Embracing the principle of nonviolence entails refraining from causing harm to fellow humans, plants, animals, and the natural world.

Source – https://egyankosh.ac.in/bitstream/123456789/35228/1/Unit-2.pdf

48. Who among the following rulers of Vijayanagara Empire constructed a large dam across Tungabhadra River and a canal-cum-aqueduct several kilometres long from the river to the capital city?

(a) Devaraya I
(b) Mallikaijuna
(c) Vira Vijaya
(d) Virupaksha

Answer – The correct answer is A, i.e Devaraya I.

Devaraya I was a prominent king of the Vijayanagara Empire who reigned from 1422 to 1446.

  • During his rule, he initiated various infrastructure projects, including the construction of a dam on the Tungabhadra River around 1410.
  • Additionally, he oversaw the building of a 24 km long aqueduct that connected the Tungabhadra River to the capital city of Vijayanagara (now called Hampi).
  • These irrigation projects played a crucial role in supporting agricultural activities and significantly contributed to the empire’s prosperity and expansion.

49. Who among the following rulers of medieval Gujarat surrendered Diu to Portuguese?

(a) Ahmad Shah
(b) Mahmud Begarha
(c) Bahadur Shah
(d) Muhammad Shah

Answer – The correct answer is C, i.e Bahadur Shah.

Bahadur Shah, the Sultan of Gujarat from 1526 to 1537, played a significant role in the history of Diu. In 1535, the Portuguese successfully captured the island of Diu from the Sultanate of Gujarat. However, it was in 1537 that Bahadur Shah agreed to a treaty with the Portuguese, formally ceding the island of Diu to them. This marked the beginning of Portuguese control over Diu, which endured for more than four centuries until it was ultimately integrated into the Indian Republic in 1961.

50. By which one of the following Acts was the Governor General of Bengal designated as the Governor General of India?

(a) The Regulating Act
(b) The Pitt’s India Act
(c) The Charter Act of 1793
(d) The Charter Act of 1833

Answer – The correct answer is D, i.e The Charter Act of 1833.

The Charter Act of 1833, also known as the Saint Helena Act, had a profound impact on the administration of British India.

One of its pivotal changes was the elevation of the Governor General of Bengal to the position of the Governor General of India.

  • This act expanded the territorial jurisdiction of the Governor General to encompass the entirety of British India.
  • Alongside this transformation, the Charter Act of 1833 introduced several other reforms of great significance.

These included the establishment of a central legislative authority, the formation of a law commission, and the promotion of education and missionary endeavors throughout India. Overall, the Charter Act of 1833 marked a crucial milestone in the governance of British India, consolidating authority and implementing key reforms for the administration of the vast territory.

51. Consider the following statements in relation to Janani Suraksha Yojana :

  1. It is a safe motherhood intervention of the State Health Departments.
  2. Its objective is to reduce maternal and neonatal mortality among poor pregnant women.
  3. It aims to promote institutional delivery among poor pregnant women.
  4. Its objective includes providing public health facilities to sick infants up to one year of age.

How many of the statements given above are correct?

(a) Only one
(b) Only two
(c) Only three
(d) All four

Answer – The correct answer is B, i.e. Only two.

JananiSurakshaYojana

Janani Suraksha Yojana (JSY) is a safe motherhood intervention under the National Rural Health Mission (NRHM) being implemented with the objective of reducing maternal and neo-natal mortality by promoting institutional delivery among the poor pregnant women. Hence statement 1 is not correct and statement 2 is correct.

JananiSurakshaYojana (JSY), a demand promotion and conditional cash transfer scheme for promoting institutional delivery. Hence statement 3 is correct.

All pregnant women belonging to the below poverty line (BPL) households and Of the age of 19 years or above Up to two live births. It is not meant for Infants. Hence statement 4 is not correct.

Source 1 – Link

52. Consider the following statements in the context of interventions being undertaken under Anaemia Mukt Bharat Strategy :

  1. It provides prophylactic calcium supplementation for pre-school children, adolescents and pregnant women.
  2. It runs a campaign for delayed cord clamping at the time of child-birth.
  3. It provides for periodic deworming to children and adolescents.
  4. It addresses non-nutritional causes of anaemia in endemic pockets with special focus on malaria, hemoglobinopathies and fluorosis.

How many of the statements given above are correct?

(a) Only one
(b) Only two
(c) Only three
(d) All four

Answer – The correct answer is C, i.e Only three.

Anameia Mukt Bharat Strategy

Prophylactic Iron Folic Acid Supplementation to all six beneficiaries age group – pre-school children, adolescents and pregnant women. Hence Statement 1 is not correct.

A promotion and monitoring campaign will be implemented to advocate for delayed clamping of the umbilical cord for newborns in all health facilities. The recommended duration for delayed clamping is at least 3 minutes, or until cord pulsations cease. This practice aims to enhance the infant’s iron reserves for up to 6 months after birth. Additionally, all birth attendants are encouraged to prioritize early initiation of breastfeeding within 1 hour of delivery. Hence statement 2 is correct.

Bi-annual mass deworming for children in the age groups between 1-19 years is carried out on designated dates – 10th February and 10th August every year under National Deworming Day (NDD) programme. The Anemia Mukt Bharat, also integrates deworming of women of reproductive age and for pregnant women as part of the NDD strategy. Hence statement 3 is correct.

The strategy attempts to intensify awareness and integrate screening and treatment for following non-nutritional causes of anemia with special focus on malaria, haemoglobinopathies and fluorosis. Hence statement 4 is correct.

Source – AnemiamukhBharat

53. Consider the following statements :

  1. Carbon fibres are used in the manufacture of components used in automobiles and aircrafts.
  2. Carbon fibres once used cannot be recycled.

Which of the statements given above is/are correct?

(a) 1 only
(b) 2 only
(c) Both 1 and 2
(d) Neither 1 nor 2

Answer – The correct answer is A, i.e 1 only.

Carbon fiber-reinforced composites find applications in various industries, including aerospace, automotive, sports equipment, and marine, where lightweight and strong materials are essential. They are utilized in the production of aircraft and spacecraft parts, racing car bodies, golf club shafts, bicycle frames, fishing rods, automobile springs, sailboat masts, and numerous other components.

One notable advantage of using carbon fiber is its positive impact on fuel efficiency. When carbon fiber composites are employed in aircraft construction, the weight reduction can reach up to 20% compared to traditional aluminum planes. This reduction in weight contributes to improved fuel efficiency, making carbon fiber an appealing choice for enhancing the performance of aircraft. Hence statement 1 is correct.

A few initiatives have been undertaken by several researchers to recycle carbon fibre waste; however, the properties of this recycled material are expected to be worse than those of virgin carbon fibre. The incorporation of polymers, nanoparticles and other hybrid materials could enhance the overall properties of recycled carbon fibre waste. Hence statement 2 is not correct.

Source 1. – NCBI

Source 2 – Dragonplate

Source 3 – IOPScience

54. Consider the following actions :

  1. Detection of car crash/collision which results in the deployment of airbags almost instantaneously
  2. Detection of accidental free fall of a laptop towards the ground which results in the immediate turning off of the hard drive
  3. Detection of the tilt of the smart-phone which results in the rotation of display between portrait and landscape mode

In how many of the above lions is the function of accelerometer required?

(a) Only one
(b) Only two
(c) All three
(d) None

Answer – The correct answer is C, i.e All three.

An accelerometer is an electronic device that measures acceleration forces experienced by an object, enabling the determination of its spatial position and the monitoring of its movement.

In the event of a car crash or collision, the accelerometer plays a crucial role by detecting the sudden deceleration of the vehicle. This detection triggers the deployment of airbags to enhance passenger safety. Hence statement 1 is correct.

Similarly, when a laptop falls, the accelerometer detects the abrupt change in motion and initiates immediate action to safeguard the hard drive. This action involves shutting down the hard drive promptly to prevent potential damage to the data stored. Hence statement 2 is correct.

Furthermore, smartphones rely on accelerometers to detect the tilt or orientation of the device. This detection enables the automatic rotation of the display between portrait and landscape modes, providing users with a more intuitive experience. Hence statement 3 is correct.

Overall, accelerometers serve crucial roles in different scenarios, from automotive safety to protecting data integrity and facilitating user-friendly interfaces on electronic devices.

Source – https://www.ncbi.nlm.nih.gov/pmc/articles/PMC9824767/#:~:text=The%20accelerometer%20sensor%20measures%20constant,s2

55. With reference to the role of biofilters in Recirculating Aquaculture System, consider the following statements :

  1. Biofilters provide waste treatment by removing uneaten fish feed.
  2. Biofilters convert ammonia present in fish waste to nitrate.
  3. Biofilters increase phosphorus as nutrient for fish in water.

How many of the statements given above are correct?

(a) Only one
(b) Only two
(c) All three
(d) None

Answer – The correct answer is B, i.e Only two.

The utilization of biofilters for the removal of contaminants from wastewater and waste gases has seen significant advancements in recent decades. Biofilters employ microorganisms that possess the capability to degrade various compounds. These microorganisms are immobilized on an inorganic or organic medium, known as a carrier, which facilitates the breakdown of pollutants present in a fluid stream. Additionally, biofilters contribute to waste treatment by effectively eliminating uneaten fish feed. Hence statement 1 is correct.

A properly designed biofilter will operate over a long period of time keeping ammonia and nitrite concentrations at levels that are not harmful to the fish. The filter’s effectiveness can be measured by the reduction in ammonia and continued low levels of nitrite in the system. The continued effectiveness of well-designed biofilters in recirculating systems can only be achieved by proper management of the system and care of the filters. Hence statement 2 is correct.

Biofilters do not contribute to an increase in phosphorus levels as a nutrient for fish in water. On the contrary, biofilters actively work towards the reduction of phosphorus (P) to minimize the risk of eutrophication in aquatic environments. The primary objective of biofilters is to remove phosphorus from the water, preventing its accumulation and subsequent negative impacts on water quality and ecosystem health. Hence statement 3 is not correct.
Source – Science Direct

56. Consider the following pairs :

Objects in space : Description

1. Cepheids : Giant clouds of dust and gas in space

2. Nebulae : Stars which brighten and dim periodically

3. Pulsars : Neutron stars that are formed when massive stars run out of fuel and collapse

How many of the above pairs are correctly matched?

  • (a) Only one
  • (b) Only two
  • (c) All three
  • (d) None

Answer – The correct answer is A, i.e Only one.

Cepheids, also called Cepheid Variables, are stars which brigthen and dim periodically. This behavior allows them to be used as cosmic yardsticks out to distances of a few tens of millions of light-years. Hence pair 1 is incorrectly matched.

A nebula is a giant cloud of dust and gas in space. Some nebulae (more than one nebula) come from the gas and dust thrown out by the explosion of a dying star, such as a supernova. Other nebulae are regions where new stars are beginning to form. For this reason, some nebulae are called “star nurseries.” Hence pair 2 is incorrectly matched.

Most neutron stars are observed as pulsars. Pulsars are rotating neutron stars observed to have pulses of radiation at very regular intervals that typically range from milliseconds to seconds. Pulsars have very strong magnetic fields which funnel jets of particles out along the two magnetic poles. These accelerated particles produce very powerful beams of light. Hence pair 3 is correctly matched.

Source – NASA

Source – SpaceplaceNASA

Source – ImaginetheUniverseNASA

57. Which one of the following countries has its own Satellite Navigation System?

(a) Australia
(b) Canada
(c) Israel
(d) Japan

Answer – The correct answer is D, i.e Japan.

Japan is the country that has its own satellite navigation system. The system is called the Quasi-Zenith Satellite System (QZSS) and it is designed to augment the existing GPS system, providing more accurate and reliable positioning and timing information within Japan and surrounding regions.

QZSS is a regional GNSS owned by the Government of Japan and operated by QZS System Service Inc. (QSS). QZSS complements GPS to improve coverage in East Asia and Oceania. Japan declared the official start of QZSS services in 2018 with 4 operational satellites, and plans to expand the constellation to 7 satellites by 2023 for autonomous capability.

Source – GPS GOV

58. Consider the following statements :

  1. Ballistic missiles are jet-propelled at subsonic speeds throughout their flights, while cruise missiles are rocket-powered only in the initial phase of flight.
  2. Agni-V is a medium-range supersonic cruise missile, while BrahMos is a solid-fuelled intercontinental ballistic missile.

Which of the statements gin above is/are correct?

(a) 1 only
(b) 2 only
(c) Both 1 and 2
(d) Neither 1 nor 2

Answer – The correct answer is D, i.e Neither 1 nor 2.

Cruise missiles are jet-propelled at subsonic speeds throughout their flights, while ballistic missiles are rocket-powered only in the initial (boost) phase of flight, after which they follow an arcing trajectory to the target. Hence statement 1 is not correct.

Agni-V is an Indian nuclear capable intercontinental ballistic missile developed by the Defence Research and Development Organization [DRDO] with a range of 5000 – 8000 Km.

The BRAHMOS missile system is a versatile supersonic cruise missile that can be deployed from various platforms, including land, sea, and air. It is a collaborative effort between DRDO (Defense Research and Development Organization) of India and NPOM (Scientific Production Association of Machine-Building) of Russia. Hence statement 2 is not correct.

Source – Britannica
Source – The Hindu

59. Consider the following statements regarding mercury pollution :

  1. Gold mining activity is a source of mercury pollution in the world.
  2. Coal-based thermal power plants cause mercury pollution.
  3. There is no known safe level of exposure to mercury.

How many of the above statements are correct?

(a) Only one
(b) Only two
(c) All three
(d) None

Answer – The correct answer is C, i.e All three.

ASGM (Artisanal and Small-Scale Gold Mining) accounts for more than a third (38%) of all anthropogenic mercury emissions to the atmosphere, with most ASGM sector-related mercury emissions coming from the burning of mercury-gold amalgam. Around the world, tens of millions of workers engage in the ASGM sector, producing up to 20% of the world’s gold annually. Hence statement 1 is correct

Mercury pollution is a well-documented issue associated with thermal power plants that rely on coal as their primary fuel source. During the process of burning coal to produce electricity, small quantities of mercury naturally found in coal can be released into the air as emissions. This release of mercury emissions from coal combustion significantly contributes to the overall pollution of mercury in the environment. Hence statement 2 is correct

Mercury is a highly toxic element; there is no known safe level of exposure. Ideally, neither children nor adults should have any mercury in their bodies because it provides no physiological benefit. Hence statement 3 is correct.

Source – United States Environmental Protection Agency
Source – NCBI

60. With reference to green hydrogen, consider the following statements :

  1. It can be used directly as a fuel for internal combustion.
  2. It can be blended with natural gas and used as fuel for heat or power generation.
  3. It can be used in the hydrogen fuel cell to run vehicles.

How many of the above statements are correct?

(a) Only one
(b) Only two
(c) All three
(d) None

Answer – The correct answer is C, All three.

Green hydrogen possesses the ability to serve as a fuel for internal combustion through utilization in a hydrogen engine (HCE). An HCE, designed to run on hydrogen (H2), functions as a modified version of a gasoline-powered engine. One significant advantage of HCEs is their emission-free nature in terms of carbon-based pollutants, making them highly advantageous in achieving carbon neutrality. HCEs operate in a manner similar to traditional 4-stroke gasoline engines (ICEs). However, hydrogen offers a higher-octane rating of 130, surpassing the octane rating of gasoline, which typically stands at 90 for premium grades. Hence statement 1 is correct.

With minor cleanup, biogas can be used to generate electricity and heat and is used as a replacement for traditional natural gas to generate combined electricity and heating for power plants—not in vehicle applications. Hence statement 2 is correct.

Fuel cell electric vehicles (FCEVs) are powered by hydrogen. They are more efficient than conventional internal combustion engine vehicles and produce no harmful tailpipe emissions—they only emit water vapor and warm air. FCEVs and the hydrogen infrastructure to fuel them are in the early stages of implementation. Hence statement 3 is correct.

Source – AFDC Energy GOV

61. Consider the following countries :

  1. Bulgaria
  2. Czech Republic
  3. Hungary
  4. Latvia
  5. Lithuania
  6. Romania

How many of the above-mentioned countries share a land border with Ukraine?

(a) Only two
(b) Only three
(c) Only four
(d) Only five

Answer – The correct answer is A, Only two.

Covering a significant expanse of land, Ukraine holds the distinction of being the largest country in Europe. Positioned in the southwestern region of the Russian Plain, it shares borders with various bodies of water, including the Black Sea and the Sea of Azov to the south. Ukraine is surrounded by Russia to the east and north, Belarus to the north, and Poland, Slovakia, and Hungary to the west. Furthermore, its southern boundaries are formed by Romania and Moldova.

62. With reference to the Earth’s atmosphere, which one of the following statements is correct?

(a) The total amount of insolation received at the equator is roughly about 10 times of that received at the poles.
(b) Infrared rays constitute roughly two-thirds of insolation.
(c) Infrared waves are largely absorbed by water vapour that is concentrated in the lower atmosphere.
(d) Infrared waves are a part of visible spectrum of electromagnetic waves of solar radiation.

Answer – The correct answer is C.

The insolation received at the surface varies from about 320 Watt/m2 in the tropics to about 70 Watt/m2 in the poles. Maximum insolation is received over the subtropical deserts, where the cloudiness is the least. Equator receives comparatively less insolation than the tropics. Hence Option a is not correct.

The atmosphere is largely transparent to short wave solar radiation. The incoming solar radiation passes through the atmosphere before striking the earth’s surface. Within the troposphere water vapour, ozone and other gases absorb much of the near infrared radiation. Hence option b is not correct and option c is correct.

Very small-suspended particles in the troposphere scatter visible spectrum both to the space and towards the earth surface. Hence option d is not correct.

Source – NCERT

63. Consider the following statements:

Statement-I :
The soil in tropical rainforests is rich in nutrients.

Statement-II :
The high temperature and moisture of tropical rainforests cause dead organic matter in the soil to decompose quickly.

Which one of the following is correct in respect of the above statement?

(a) Both Statement-I and Statement-II are correct and Statement-II is the correct explanation for Statement-I
(b) Both Statement-I and Statement-II are correct and Statement-II is not the correct explanation for Statement-I
(c) Statement-1 is correct but Statement-II is incorrect
(d) Statement-1 is incorrect but Statement-II is correct

Answer – The correct answer is D.

Statement 1 is incorrect: Contrary to the statement, tropical rainforest soil is not necessarily poor in nutrients required by plants to grow.

  • While it is true that the acidic nature of the soil and frequent rainfall can result in leaching, causing the loss of nutrients such as potassium, calcium, magnesium, and phosphorus, it does not imply that the soil is inherently lacking in nutrients.
  • The nutrient content of tropical rainforest soil can vary depending on various factors, including the specific location and soil composition.
  • Furthermore, while regeneration may be slower in tropical rainforests compared to other ecosystems, it does occur through natural processes.

Statement 2 is correct: Tropical rainforests indeed experience warm and consistent temperatures throughout the year.

  • These elevated temperatures promote the activity of soil microorganisms responsible for decomposing organic matter.
  • The increased microbial activity leads to faster rates of decomposition.
  • Additionally, tropical rainforests receive abundant rainfall, resulting in high levels of soil moisture.
  • Moisture is essential for facilitating microbial activity and the breakdown of organic matter.
  • It creates an environment conducive to enzymatic processes involved in decomposition and provides a suitable habitat for decomposers like bacteria and fungi.

64. Consider the following statements :

Statement-I :
The temperature contrast between continents and oceans is greater during summer than in winter.

Statement-II :
The specific heat of water is more than that of land surface.

Which one of the following is correct in respect of the above statements?

(a) Both Statement-I and Statement-II are correct and Statement-II is the correct explanation for Statement-I
(b) Both Statement-I and Statement-II are correct and Statement-II is not the correct explanation for Statement-I
(c) Statement-1 is correct but Statement-II is incorrect
(d) Statement-1 is incorrect but Statement-II is correct

Answer – The correct answer is D.

Statement 1 is incorrect: The temperature contrast between continents and oceans is greater in winter, not summer.

Statement 2 is correct: Water has a high specific heat, requiring more heat energy to raise its temperature compared to land.

Explanation: The higher specific heat of water (Statement 2) explains the temperature contrast between continents and oceans during summer (Statement 1). Land heats up quickly due to its lower specific heat, causing rapid temperature increases in summer. On the other hand, water’s higher specific heat leads to a slower temperature increase, resulting in a temperature contrast between land and ocean during summer.

65. Consider the following statements :

  1. In a seismograph, P waves are recorded earlier than S waves.
  2. In P waves, the individual particles vibrate to and fro in the direction of wave propagation whereas in S waves, the particles vibrate up and down at right angles to the direction of wave propagation.

Which of the statements given above is/are correct?

(a) 1 only
(b) 2 only
(c) Both 1 and 2
(d) Neither 1 nor 2

Answer – The correct answer is C, i.e Both 1 and 2.

Statement 1 is correct: P-waves, also known as primary waves, are the first to arrive at the surface.

Statement 2 is correct: P-waves exert pressure in the direction of propagation, while S-waves create troughs and crests perpendicular to the wave direction. Surface waves are the most damaging.

Explanation: Earthquake waves consist of body waves and surface waves. Body waves are generated at the earthquake’s focus and move through the body of the Earth, interacting with surface rocks and generating surface waves. P-waves, or primary waves, arrive first and vibrate parallel to the wave direction, exerting pressure on the material. S-waves, or secondary waves, arrive later and vibrate perpendicular to the wave direction, creating troughs and crests. Surface waves are the most destructive type of waves in earthquakes.

66. With reference to coal-based thermal power plants in India, consider the following statements :

  1. None of them uses seawater.
  2. None of them is set up in water-stressed district.
  3. None of them is privately owned.

How many of the above statements are correct?

  • (a) Only one
  • (b) Only two
  • (c) All three
  • (d) None

The correct answer is D.

Statement 1 is incorrect: The Mundra Ultra Mega Power Project (UMPP) located in Tunda village, Kutch district, Gujarat, India, is a sub-bituminous coal-fired power plant. It ranks as the third largest operational power plant in India. The primary source of coal for this power plant is imported from Indonesia. Furthermore, the plant utilizes seawater from the Gulf of Kutch as its water source.

Statement 2 is incorrect: Contrary to the statement, 40% of India’s Thermal Power Plants are situated in regions that face water scarcity.

Statement 3 is incorrect: Contrary to the statement, India has a total of 269 Thermal Power Plants. Among these, 138 are owned by the public sector, while the remaining 131 are owned by the private sector.

Source

https://www.wri.org/insights/40-indias-thermal-power-plants-are-water-scarce-areas-threatening-shutdowns

https://cea.nic.in/wp-content/uploads/pdm/2021/06/list_power_stations_2021.pdf

67. Wolbachia method’ is sometimes talked about with reference to which one of the following?

(a) Controlling the viral diseases spread by mosquitoes
(b) Converting crop residues into packing material
(c) Producing biodegradable plastic
(d) Producing biochar from thermochemical conversion of biomass

Answer – The correct answer is A.

The “Wolbachia method” is an effective technique utilized to control the spread of viral diseases transmitted by mosquitoes, including dengue, Zika, and chikungunya. This method involves introducing Wolbachia, a type of bacterium, into mosquitoes to limit their ability to transmit these diseases. By mating with mosquitoes lacking Wolbachia, the resulting eggs either fail to hatch or do not produce viable offspring, leading to a reduction in the population of disease-carrying mosquitoes.

Source: The Hindu

68. Consider the following activities :

  1. Spreading finely ground basalt rock on farmlands extensively
  2. Increasing the alkalinity of oceans by adding lime
  3. Capturing carbon dioxide released by various industries and pumping it into abandoned subterranean mines in the form of carbonated waters

How many of the above activities are often considered and discussed for carbon capture and sequestration?

(a) Only one
(b) Only two
(c) All three
(d) None

The correct answer is C, i.e All three.

Statement 1 is correct: Enhanced weathering involves the extensive spreading of finely ground basalt rock on farmlands. This process accelerates the natural capture of carbon dioxide from the atmosphere as the basalt reacts with CO2 to form stable carbonates, effectively storing carbon.

Statement 2 is correct: Increasing the alkalinity of oceans through the addition of lime, also known as ocean alkalinity enhancement, involves introducing lime (calcium oxide or calcium hydroxide) to the oceans. This method enhances the alkalinity of the oceans, enabling them to absorb and store more carbon dioxide from the atmosphere, thus aiding in carbon sequestration.

Statement 3 is correct: Carbon capture and storage (CCS) involves capturing the carbon dioxide emitted by various industries and storing it underground. Abandoned subterranean mines can serve as suitable storage sites, where the captured carbon dioxide is pumped into these locations in the form of carbonated waters. This process facilitates the secure storage of carbon dioxide emissions.

Source –

69. ‘Aerial metagenomics’ best refers to which one of the following situations?

(a) Collecting DNA samples from air in a habitat at one go
(b) Understanding the genetic makeup of avian species of a habitat
(c) Using air-borne devices to collect blood samples from moving animals
(d) Sending drones to inaccessible areas to collect plant and animal samples from land surfaces and water bodies

Answer – The correct answer is A.

Aermal metagenomics” is the term used to describe the process of collecting and examining DNA samples obtained from the air or aerosol particles within a specific habitat. This scientific method involves sampling the air in a particular environment to capture the genetic material (DNA) of microorganisms such as bacteria, viruses, fungi, and other microscopic organisms.

70. `Microsatellite DNA’ is used in the case of which one of the following?

(a) Studying the evolutionary relationships among various species of fauna
(b) Stimulating ‘stem cells’ to transform into diverse functional tissues
(c) Promoting clonal propagation of horticultural plants
(d) Assessing the efficacy of drugs by conducting series of drug trials in a population

Answer – The correct answer is A.

Microsatellite DNA, also referred to as short tandem repeats (STRs), is a specific kind of genetic marker composed of repetitive DNA sequences. It is widely employed in genetic research, such as population genetics and phylogenetic analysis, to investigate the connections between various animal species. Microsatellites exhibit high polymorphism and their lengths differ due to variations in the number of repeated units. Through the analysis of these variations in microsatellite DNA sequences among individuals or populations, scientists can evaluate genetic diversity, population structure, and evolutionary relationships within and across species.

71. Consider the following statements with reference to India :

  1. According to the ‘Micro, Small and Medium Enterprises Development (MSMED) Act, 2006’, the ‘medium enterprises’ are those with investments in plant and machinery between Rs 15 crore and Rs 25 crore.
  2. All bank loans to the Micro, Small and Medium Enterprises qualify under the priority sector.

Which of the statements given above is/are correct?

(a) 1 only
(b) 2 only
(c) Both 1 and 2
(d) Neither 1 nor 2

Answer – The correct answer is D.

Statement 1 is incorrect: According to the Micro, Small, and Medium Enterprises Development (MSMED) Act of 2006, medium enterprises are defined as those with investments in Plant and Machinery not exceeding Rs. 50 crore and turnover not exceeding Rs. 250 crore.

Statement 2 is incorrect: Not all bank loans to MSMEs meeting the criteria outlined in the Master Direction on “Priority Sector Lending (PSL) – Targets and Classification” dated September 4, 2020, are eligible to be classified as priority sector lending.

72. With reference to Central Bank digital currencies, consider the following statements :

  1. It is possible to make payments in a digital currency without using US dollar or SWIFT system.
  2. A digital currency can be distributed with a condition programmed into it such as a time frame for spending it.

Which of the statements given above is/are correct?

(a) 1 only
(b) 2 only
(c) Both 1 and 2
(d) Neither 1 nor 2

Answer – The correct answer is C, i.e Both 1 and 2.

Statement 1 is incorrect: According to the Micro, Small, and Medium Enterprises Development (MSMED) Act of 2006, medium enterprises are defined as those with investments in Plant and Machinery not exceeding Rs. 50 crore and turnover not exceeding Rs. 250 crore.

Statement 2 is incorrect: Not all bank loans to MSMEs meeting the criteria outlined in the Master Direction on “Priority Sector Lending (PSL) – Targets and Classification” dated September 4, 2020, are eligible to be classified as priority sector lending.

73. In the context of finance, the term ‘beta’ refers to

(a) the process of simultaneous buying and selling of an asset from different platforms
(b) an investment strategy of a portfolio manager to balance risk versus reward
(c) a type of systemic risk that arises where perfect hedging is not possible
(d) a numeric value that measures the fluctuations of a stock to changes in the overall stock market

Answer – The correct answer is A.

Beta is a measure that quantifies a stock’s expected movement in relation to overall market fluctuations. It provides insights into the stock’s volatility relative to the broader market. A beta greater than 1 suggests that the stock is more volatile than the market, while a beta less than 1 indicates lower volatility.

The Capital Asset Pricing Model (CAPM) is a financial model that evaluates a stock’s potential return and utilizes beta as a key component in its calculation. By incorporating beta, the model helps assess the stock’s level of volatility and systematic risk.

Source: The Economic Times

74. Consider the following statements :

  1. The Self-Help Group (SHG) Programme was originally initiated pr State Bank of India by providing microcredit to the financially deprived.
  2. In an SHG, all members of a group take responsibility for a loan that an individual member takes.
  3. The Regional Rural Banks and Scheduled Commercial Banks support SHGs.

How many of the above statements are correct?

(a) Only one
(b) Only two
(c) All three
(d) None

Answer – The correct answer is B, i.e Only two.

Statement 1 is incorrect: NABARD initiated the promotion of self-help groups in 1991-1992, which played a crucial role in launching the “SHG movement.” In 1993, the Reserve Bank of India granted permission for SHGs to open savings bank accounts, significantly boosting the movement by providing access to banking services.

Statement 2 is correct: Banks provide loans to self-help groups as a collective entity, and it becomes the collective responsibility of the SHG members to repay the loan. Therefore, all members of an SHG share the responsibility for loans taken by individual members. Additionally, the SHG may decide not to provide further loans to a member who defaults on repayment.

Statement 3 is correct: The Financial Inclusion Fund scheme aims to support Scheduled Commercial Banks (SCBs) and Regional Rural Banks (RRBs) in enabling the “Dual Authentication” feature in microATMs for SHG transactions. This scheme creates an enabling ecosystem for SHGs to seamlessly operate at Business Correspondent points. SCBs and RRBs provide support to SHGs in accessing credits according to their needs.

Sources: NCW Maharashtra, NABARD Circular

75. Consider the following statements :

Statement-I :
India’s public sector health care system largely focuses on curative care with limited preventive, promotive and rehabilitative care.

Statement-II:
Under India’s decentralized approach to health care delivery, the States are primarily responsible for organizing health services.

Which one of the following is correct in respect of the above statements?

(a) Both Statement-I and Statement-II are correct and Statement-II is the correct explanation for Statement-I
(b) Both Statement-I and Statement-II are correct and Statement-II is not the correct explanation for Statement-I
(c) Statement-I is correct but Statement-II is incorrect
(d) Statement-I is incorrect but Statement-II is correct

Answer – The correct answer is D.

Statement 1 is incorrect: Health care encompasses not only medical care but also includes preventive, curative, and rehabilitative care, all of which are given significant importance.

Statement 2 is correct: In India’s decentralized approach to health-care delivery, the responsibility for organizing health services primarily lies with the states. Public health is listed as a subject under the State list of the 7th Schedule, highlighting the states’ role in healthcare.

76. Consider the following statements:

Statement-I :
According to the United Nations’ ‘World Water Development Report, 2022’ India extracts more than a quarter of the world’s groundwater withdrawal each year.

Statement-II :
India needs to extract more than a quarter of the world’s groundwater each year to satisfy the drinking water and sanitation needs of almost 18% of world’s population living in its territory.

Which one of the following is correct in respect of the above statements?

(a) Both Statement-I and Statement-II are correct and Statement-II is the correct explanation for Statement-I
(b) Both Statement and Statement-I and Statement-II are correct and Statement-II is not the correct explanation for Statement-I
(c) Statement-I is correct but Statement-II is incorrect
(d) Statement-I is incorrect but Statement-II is correct

The correct answer is C.

Statement 1 is correct: According to the United Nations World Water Development Report 2022, India is the largest user of groundwater globally. With an estimated annual withdrawal of 251 km3 from approximately 20 million wells and tube wells, India accounts for over a quarter of the total groundwater extracted internationally.

Statement 2 is incorrect: The majority of the groundwater extracted in India, approximately 89%, is used for irrigation purposes rather than for drinking and sanitation purposes.

Source: United Nations World Water Development Report 2022

77. Consider the following statements :

  1. According to the Constitution of India, the Central Government has a duty to protect States from internal disturbances.
  2. The Constitution of India exempts the States from providing legal counsel to a person being held for preventive detention.
  3. According to the Prevention of Terrorism Act, 2002, confession of the accused before the police cannot be used as evidence.

How many of the above statements are correct?

(a) Only one
(b) Only two
(c) All three
(d) None

Answer – The correct answer is A, i.e Only one.

Statement 1 is correct: Article 355 of the Constitution of India mandates the Union’s responsibility to protect every state against external aggression and internal disturbance and ensure that the government of every state adheres to the provisions of the Constitution.

Statement 2 is incorrect: Article 22(1) of the Constitution of India explicitly states that any person who is arrested shall have the right to be informed of the grounds for their arrest and the right to consult and be defended by a legal practitioner of their choice. Article 22(5) further states that if a person is detained under a preventive detention law, the authority responsible for the detention must promptly communicate the grounds for the order and provide the detainee with an opportunity to make a representation against the order.

Statement 3 is incorrect: The Prevention of Terrorism Act, 2002, does not explicitly mention that confessions made by the accused before the police cannot be used as evidence. However, Section 16 of the Act stipulates that confessions made by individuals to a police officer are inadmissible in any legal proceeding against them. To be admissible as evidence, a confession made to the police must be supported by corroborating evidence.

Therefore, only statement 1 is correct.

78. Which one of the following countries has been suffering from decades of civil strife and food shortages and was in news in the recent past for its very severe famine?

(a) Angola
(b) Costa Rica
(c) Ecuador
(d) Somalia

Answer – The correct answer is Somalia.

Due to a prolonged drought and the consequences of three decades of civil unrest, Somalia has reached the highest position on the International Rescue Committee’s Emergency Watchlist for the first time in history. With five consecutive seasons of failed rainfall, the country is currently facing extreme food insecurity, affecting 6.5 million people amidst the driest conditions witnessed in four decades. The compounding impact of recurrent climate-related disasters, persistent insecurity, and continued instability is exacerbating the already dire situation caused by the drought.

79. Consider the following statements :

  1. In India, the Biodiversity Management Committees are key to the realization of the objectives of the Nagoya Protocol.
  2. The Biodiversity Management Committees have important functions in determining access and benefit sharing, including the power to levy collection fees on the access of biological resources within its jurisdiction.

Which of the statements given above is/are correct?

(a) 1 only
(b) 2 only
(c) Both 1 and 2
(d) Neither 1 nor 2

Answer – The correct answer is C, i.e Both 1 and 2.

Statement 1 is correct: The Nagoya Protocol is an international agreement linked to the Convention on Biological Diversity. Its purpose is to ensure the fair and equitable sharing of benefits arising from the utilization of genetic resources. The protocol came into effect on October 12, 2014.

In accordance with the Biological Diversity Act of 2002, every local body in India is required to establish a Biodiversity Management Committee (BMC) within its jurisdiction. The primary objective of these committees is to promote the conservation, sustainable use, and documentation of biological diversity. In India, BMCs serve as the implementing agency for the objectives outlined in the Nagoya Protocol.

Statement 2 is correct: The Biological Diversity Act grants BMCs the authority to charge collection fees to individuals who gather biological resources within their jurisdiction. This provision enables BMCs to generate their own financial resources, which can be utilized to fulfill the objectives outlined in the Act.

80. Consider the following statements in respect of election to the President of India :

  1. The members nominated to either House of the Parliament or the Legislative Assemblies of States are also eligible to be included in the Electoral College.
  2. Higher the number of elective Assembly seats, higher is the value of vote of each MLA of that State..
  3. The value of vote of each MLA of Madhya Pradesh is greater than that of Kerala.
  4. The value of vote of each MLA of Puducherry is higher than that of Arunachal Pradesh because the ratio of total population to total number of elective seats in Puducherry is greater as compared to Arunachal Pradesh.

How many of the above statement are correct ?

(a) Only one
(b) Only two
(c) Only three
(d) All four

Answer – The correct answer is A, i.e Only one.

Statement 1 is incorrect: The electoral college for the Presidential Election in India consists of elected members (not nominated) of the Rajya Sabha (Upper House), Lok Sabha (Lower House), and the Legislative Assemblies of States and Union Territories (MLAs).

Statement 2 is incorrect: The value of each MLA’s vote in the Presidential Election is determined by dividing the State’s population by the number of elected members in its legislative assembly, and then further dividing the quotient by 1,000. The calculation takes into account each State’s population and the number of members in its legislative assembly. However, the population data used for this calculation is not based on the 1971 Census as mentioned but is updated periodically.

Statement 3 is incorrect: The vote value of each MLA in Madhya Pradesh cannot be compared directly to that of Kerala based solely on the population-to-elective seat ratio. The vote value calculation takes into account the specific population and the number of elected members in each State’s legislative assembly.

Statement 4 is correct: The vote value of each MLA in Puducherry is higher than that of Arunachal Pradesh due to the difference in the population-to-elective seat ratio. In the 2022 Presidential election, the vote value of each MLA from Puducherry was 16, whereas the vote value of each MLA from Arunachal Pradesh was 8.

81. With reference to the Indian History, Alexander Rea, A. H. Longhurst, Robert Sewell, James Burgess and Walter Elliot were associated with

(a) archaeological excavations
(b) establishment of English Press in Colonial India
(c) establishment of Churches in Princely States
(d) construction of railways in Colonial India

Answer – The correct answer is A, i.e Archaeological excavations.

All the mentioned individuals have played significant roles in archaeological excavations during the British rule in India.

Option a is correct: Alexander Rea (1858-1924) was a British archaeologist who primarily worked in South India. He is renowned for his discovery of a sarcophagus in the hillocks of Pallavaram in Tamil Nadu.

Albert Henry Longhurst (1876 – 1955) was a British archaeologist and art historian who conducted excavations in India and Ceylon. He led the systematic dig at Nagarjunakonda from 1927 to 1931.

Robert Sewell (1845–1925) served in the civil service of the Madras Presidency during the colonial era in India. Sewell engaged in archaeological work at the Amaravati Buddhist stupa, which had already suffered significant damage prior to his involvement. The site had been previously surveyed by Colin Mackenzie and Walter Elliot (1803-1887).

James Burgess (1832 – 1916) was a prominent archaeologist of British India in the 19th century and the founder of the Indian Antiquary in 1872. From 1886 to 1889, he held the position of Director General of the Archaeological Survey of India.

82. Consider the following pairs :

Site : Well known for

1. Besnagar : Shaivite cave shrine

2. Bhaja : Buddhist cave shrine

3. Sittanavasal : Jain cave shrine

How many of the above pairs are correctly matched?

  • (a) Only one
  • (b) Only two
  • (c) All three
  • (d) None

Answer – The correct answer is B, i.e Only two.

Pair 1 is incorrect: Besnagar, also known as Vidisha, is a town in modern Madhya Pradesh. The Heliodorus Pillar, located on the outskirts of Besnagar, is dedicated to the Hindu god Vishnu. It was erected by a Greek man named Heliodorus, who is believed to have embraced Hinduism and become a disciple of Lord Vishnu. The pillar, known locally as “Khamba Baba,” is revered by the community. Therefore, Besnagar is associated with the Vaishnavite sect, not the Shaivite sect.

Pair 2 is correct: The Bhaja Caves, also known as Bhaje Caves, are a group of 22 rock-cut caves dating back to the 2nd century BC. They are located in the Pune district near Lonavala, Maharashtra. These caves are associated with the Hinayana Buddhism sect in Maharashtra and contain numerous stupas. The most significant excavation in the caves is the chaitya griha, specifically Cave XII.

Pair 3 is correct: The Sithannavasal caves, situated in the Pudukottai District of modern Tamil Nadu, are associated with Jainism. In ancient times, Jain monks inhabited these caves and hills to practice their ascetic lives. They carved the hills for rituals and penance in open shelters.

83. Consider the following statements :

Statement- I:
7th August declared as the National Handloom Day.

Statement-II
It was in 1905 that the Swadeshi Movement was launched on the same day.

Which one of the following is correct in respect of the above statements?

(a) Both Statement-I and Statement-II are correct and Statement-II is the correct explanation for Statement-I
(b) Both Statement and Statement-I and Statement-II are correct and Statement-II is not the correct explanation for Statement-I
(c) Statement-I is correct but Statement-II is incorrect
(d) Statement-I is incorrect but Statement-II is correct

Answer – The correct answer is A.

The first National Handloom Day took place on August 7, 2015, as a way to honor the handloom weaving community and recognize their significant contribution to the socio-economic development of the country.

Statement 1 is correct: The National Handloom Day is an annual observance held on August 7 to pay tribute to the handloom weavers in India and to highlight the importance of the handloom industry in the country.

Statement 2 is correct: August 7 was chosen as the date for National Handloom Day to commemorate the Swadeshi Movement, which was initiated on August 7, 1905, at Calcutta Town Hall. The movement aimed to protest against the partition of Bengal by the British Government and promote the use of domestic products and production methods. Therefore, Statement 2 provides the correct explanation for Statement 1.

84. Consider the following statements in respect of the National Flag of India according to the Flag Code of India, 2002 :

Statement-I :
One of the standard sizes of the National Flag of India is 600 mm x 400 mm.

Statement-II :
The ratio of the length to the height (width) of the Flag shall be 3 : 2.

Which one of the following is correct in respect of the above statements?

(a) Both Statement-I and Statement-II are correct and Statement-II is the correct explanation for Statement-I
(b) Both Statement and Statement-I and Statement-II are correct and Statement-II is not the correct explanation for Statement-I
(c) Statement-I is correct but Statement-II is incorrect
(d) Statement-I is incorrect but Statement-II is correct

Answer – The correct answer is D.

The use, display, and hoisting of the National Flag in India are governed by a comprehensive set of guidelines known as the ‘Flag Code of India 2002.’ This code encompasses all the laws, conventions, practices, and instructions related to the proper display of the National Flag.

Statement 1 is incorrect: According to the Flag Code of India 2002, there is no specific restriction on the dimensions of the National Flag being 600mm*400mm. Instead, the code defines nine standard dimensions for the National Flag.

Statement 2 is correct: The National Flag of India should have a rectangular shape, and the ratio of its length to height (width) should be 3:2. This aspect is in accordance with the Flag Code.

Source: https://www.mha.gov.in/sites/default/files/flagcodeofindia_070214.pdf (Page 3)

85. Consider the following statements in respect of the Constitution Day :

Statement-I :
The Constitution Day is celebrated on 26th November every year to promote constitutional values among citizens.

Statement-II :
On 26th November, 1949, the Constituent Assembly of India set up a Drafting Committee under the Chairmanship of Dr. B. R. Ambedkar to prepare a Draft Constitution of India.

Which one of the following is correct in respect of the above statements?

(a) Both Statement-I and Statement-II are correct and Statement-II is the correct explanation for Statement-I
(b) Both Statement-I and Statement-II are correct and Statement-II is not the correct explanation for Statement-I
(c) Statement-I is correct but Statement-II is incorrect
(d) Statement-I is incorrect but Statement-II is correct

Answer – The correct answer is C.

Statement 1 is correct: Constitution Day is celebrated in India on 26th November every year to commemorate the adoption of the Constitution of India and to promote constitutional values among the citizens.

Statement 2 is incorrect: On 29th August 1947, the Constituent Assembly set up a Drafting Committee chaired by Dr. B.R. Ambedkar to prepare the Draft Constitution for India. The Constituent Assembly of India adopted the Constitution on 26th November 1949, and it came into effect on 26th January 1950.

Source: https://www.mygov.in/campaigns/constitutionday/#:~:text=Constitution%20Day%27%20to-,promote,-Constitution%20values%20among

86. Consider the following statements:

Statement-1:
Switzerland is one of the leading exporters of gold in terms of value.

Statement-II :
Switzerland has the second largest gold reserves in the world.

Which one of the following is correct in respect of the above statements?

(a) Both Statement-I and Statement-II are correct and Statement-II is the correct explanation for Statement-I
(b) Both Statement-1 and Statement-II are correct and Statement-II is not the correct explanation for Statement-1
(c) Statement-I is correct but Statement-II is incorrect
(d) Statement-I is incorrect but Statement-II is correct

The correct answer is C.

Statement 1 is correct: Switzerland is indeed the leading exporter of gold in the world. In 2021, Switzerland exported $86.7 billion worth of gold, making it the largest exporter of gold globally. The main destinations for Switzerland’s gold exports are India ($29.3 billion), China ($16 billion), the United States ($8.13 billion), Germany ($5.8 billion), and Hong Kong ($4.67 billion).

Statement 2 is incorrect: Switzerland does not have the second largest gold reserves in the world; it actually has the seventh largest gold reserve. The top three countries with the largest gold reserves are the United States of America (USA), followed by Germany and Italy.

Source: Data on Swiss gold exports – https://oec.world/en/profile/country/che#Exports

87. Consider the following statements:

Statement-I :
Recently, the United States of America (USA) and the European Union (EU) have launched the ‘Trade and Technology Council’

Statement-II :
The USA and the EU claim that through this they are trying to bring technological progress and physical productivity under their control.

Which one of the following is correct in respect of the above statements?

(a) Both Statement-I and Statement-II are correct and Statement-II is the correct explanation for Statement-I
(b) Both Statement-I and Statement-II are correct and Statement-II is not the correct explanation for Statement-I
(c) Statement-I is correct but Statement-II is incorrect
(d) Statement-1 is incorrect but Statement-II is correct

Answer – The correct answer is C.

Statement 1 is correct: The Trade and Technology Council was indeed launched by the United States of America and the European Union during the EU-US Summit on 15 June 2021 in Brussels.

Statement 2 is incorrect: The Trade and Technology Council established by the USA and EU does not seek to exert control over technological progress and physical productivity. Instead, the initiative aims to foster collaboration and cooperation in areas such as digital resources, including AI models and computing power. The goal is to share these resources with partner countries to address various challenges, including climate change, extreme weather events, healthcare, and smart agriculture.

Source: Joint Statement of the EU-US Summit – https://www.consilium.europa.eu/en/press/press-releases/2021/06/15/eu-us-summit-joint-statement/

88. Consider the following statements :

Statement-I :
India accounts for 3.2% of global export of goods.

Statement-II :
Many local companies and some foreign companies operating in India have taken advantage of India’s ‘Production-linked Incentive’ scheme.

Which one of the following is correct in respect of the above statements?

(a) Both Statement-I and Statement-II are correct and Statement-II is the correct explanation for Statement-I
(b) Both Statement-I and Statement-II are correct and Statement-II is not the correct explanation for Statement-I
(c) Statement-I is correct but Statement-II is incorrect
(d) Statement-1 is incorrect but Statement-II is correct.

Answer – The correct answer is D.

Statement 1 is incorrect: According to the latest available data from the Economic Survey 2022-23, India accounts for 1.8% of global exports of goods and 4% of global exports of services. Therefore, the statement that India accounts for 3.2% of global exports of goods is not correct.

Statement 2 is correct: The Production Linked Incentive (PLI) scheme is a performance-linked incentive program provided to companies based on their incremental sales from products manufactured in domestic units. Its objective is to boost the manufacturing sector and reduce imports in India. The PLI scheme is available to both domestic and foreign companies operating in India, and many such companies have indeed taken advantage of these schemes.

Source: Economic Survey 2022-23 (page 315)

89. Consider the following statements :

The ‘Stability and Growth Pact’ of the European Union is a treaty that

  1. limits the levels of the budgetary deficit of the countries of the European Union
  2. makes the countries of the European Union to share their infrastructure facilities
  3. enables the countries of the European Union to share their technologies

How many of the above statements are correct?

(a) Only one
(b) Only two
(c) All three
(d) None

Answer – The correct answer is A, i.e Only one.

Statement 1 is correct: The Stability and Growth Pact (SGP) is an agreement among all 27 member states of the European Union. Its purpose is to promote and maintain the stability of the Economic and Monetary Union (EMU). The SGP involves fiscal monitoring of member states by the European Commission and the Council of the European Union. It also includes issuing yearly recommendations for policy actions to ensure compliance with the SGP in the medium-term. The SGP is primarily based on Articles 121 and 126 of the Treaty on the Functioning of the European Union.

Statements 2 and 3 are incorrect: The SGP does not involve the sharing of infrastructure facilities or technologies among countries in the European Union. The focus of the SGP is primarily on fiscal policies and economic stability rather than technology or infrastructure sharing.

90. Consider the following statements:

  1. Recently, all the countries of the United Nations have adopted the first-ever compact for international migration, the ‘Global Compact for Safe, Orderly and Regular Migration (GCM)’.
  2. The objectives and commitments stated in the GCM are binding on the UN member countries.
  3. The GCM addresses internal migration or internally displaced people also in its objectives and commitments.

How many of the above statements are correct?

(a) Only one
(b) Only two
(c) All three
(d) None

The correct answer is A, i.e Only one.

The Global Compact for Safe, Orderly, and Regular Migration is an intergovernmental agreement developed within the framework of the United Nations. It represents the first comprehensive and holistic international agreement on migration.

Statement 1 is incorrect: Currently, the Global Compact for Safe, Orderly, and Regular Migration (GCM) has been adopted by 163 countries. While some countries, including the United States, Hungary, Israel, Czech Republic, and Poland, voted against the compact, it has received broad support from a majority of UN member states.

Statement 2 is incorrect: The Global Compact for Migration is a non-legally binding agreement. It is rooted in principles of state sovereignty and responsibility-sharing, recognizing the need for cooperation to maximize the overall benefits of migration.

Statement 3 is correct: The global compact consists of 23 objectives aimed at improving the management of migration at various levels: local, national, regional, and global. These objectives demonstrate the commitment to address both internal and international migration challenges.

Sources:

91. With reference to Home Guards, consider the following statements

  1. Home Guards are raised under the Home Guards Act and Rules of the Central Government.
  2. The role of the Home Guards is to serve as an auxiliary force to the police in maintenance of internal security.
  3. To prevent infiltration on the international border/coastal areas, the Border Wing Home Guards Battalions have been raised in some States.

How many of the above statements are correct?

(a) Only one
(b) Only two
(c) All three
(d) None

Answer – The correct answer is B, i.e Only two.

Home Guards‘ is a voluntary force that was initially established in India in December 1946 to support the police in maintaining control during civil disturbances and communal riots.

Statement 1 is incorrect: Home Guards are raised under the Home Guards Act and Rules of the respective States/Union Territories, not the central government. They recruit individuals from various backgrounds and walks of life who volunteer their spare time for the betterment of the community.

Statement 2 is correct: The primary role of Home Guards is to serve as an auxiliary force to the police in handling internal security situations. They assist the community in emergencies such as air raids, fires, cyclones, earthquakes, and epidemics. They also contribute to the maintenance of essential services, promote communal harmony, assist the administration in protecting vulnerable sections of society, engage in socio-economic and welfare activities, and perform civil defense duties.

Statement 3 is correct: To prevent infiltration on the international border and coastal areas, guard vital areas and points, and secure communication lines during external aggression, fifteen Border Wing Home Guards (BWHG) Battalions have been established in border states. These include six battalions in Punjab, four in Rajasthan, two in Gujarat, and one each in Meghalaya, Tripura, and West Bengal. They function as an auxiliary to the Border Security Force.

Therefore, only two statements are correct.

Source: https://dgfscdhg.gov.in/about-homeguard

92. With reference to India, consider the following pairs:

Action : The Act under which it is covered

1. Unauthorized wearing: of police or military uniforms : The Official Secrets Act, 1923

2. Knowingly misleading or otherwise interfering with a police officer or military officer when engaged in their duties. : The Indian Evidence Act, 1872

3. Celebratory gunfire which can endanger the personal safety of others. : The Arms (Amendment) Act, 2019

How many of the above pairs are correctly matched?

(a) Only one
(b) Only two
(c) All three
(d) None

Answer – The correct answer is Only two.

Pair 1 is correctly matched: The Official Secrets Act, 1923 prohibits individuals from using or wearing any naval, military, air force, police, or other official uniform without lawful authority. It also prohibits falsely representing oneself as a person entitled to wear such a uniform.

Pair 2 is incorrectly matched: The provision mentioned does not fall under The Official Secrets Act, 1923. It is not related to obstructing or interfering with police officers or members of the armed forces engaged in guard or patrol duties near a prohibited place.

Pair 3 is correctly matched: The Arms Act Amendment of 2019 introduced new offenses, including forcefully taking a firearm from the police or armed forces and using firearms in celebratory gunfire that endangers human life or personal safety.

Source:

93. Consider the following pairs:

Regions often mentioned in news : Reason for being in news

1. North Kivu and Ituri : War between Armenia and Azerbaijan.

2. Nagorno-Karabakh : Insurgency in Mozambique.

3. Kherson and Zaporizhzhia : Dispute between Israel and Lebanon.

How many of the above pairs are correctly matched?

(a) Only one
(b) Only two
(c) All three
(d) None

The correct answer is D, i.e None.

Pair 1 is not correctly matched: North Kivu and Ituri are indeed provinces of the Democratic Republic of the Congo. However, the statement about a humanitarian emergency in North Kivu, Ituri, and South Kivu in 2023 is not supported by the given information.

Pair 2 is not correctly matched: The statement accurately describes the unresolved dispute over the landlocked mountainous region of Nagorno-Karabakh, but is is located within Azerbaijan but has an ethnic Armenian majority and has been supported by neighboring Armenia.

Pair 3 is not correctly matched: The statement incorrectly claims that Russia carried out referendums on joining Russia in the Ukrainian regions of Luhansk, Donetsk, Zaporizhzhia, and Kherson. These regions, particularly Luhansk and Donetsk, have experienced conflicts and separatist movements but have not officially joined Russia through referendums.

94. Consider the following statements:

Statement-1:
Israel has established diplomatic relations with some Arab States.

Statement-II :
The ‘Arab Peace Initiative’ mediated by Saudi Arabia was signed by Israel and Arab League.

Which one of the following is correct in respect of the above statements?

(a) Both Statement-I and Statement-II are correct and Statement-II is the correct explanation for Statement-I
(b) Both Statement-I and Statement-II are correct and Statement-II is not the correct explanation for Statement-1
(c) Statement-I is correct but Statement-II is incorrect
(d) Statement-I is incorrect but Statement-II is correct

Answer – The correct answer C.

Statement 1 is correct: Israel has indeed established diplomatic relations with some Arab countries over the years. The first Arab country to officially recognize and establish diplomatic relations with Israel was Egypt. The peace treaty between Israel and Egypt was signed in 1979 at Camp David, leading to diplomatic relations and normalization of ties between the two nations.

Another Arab country that has established diplomatic relations with Israel is Jordan. In 2020, the United Arab Emirates (UAE) and Bahrain also signed agreements with Israel to establish full diplomatic relations, known as the Abraham Accords.

Statement 2 is incorrect: The Arab Peace Initiative was not signed by Israel. The initiative, proposed by Saudi Arabia in 2002 at the Arab League Summit in Beirut, called for the recognition of Israel by the Arab states and the establishment of normal diplomatic relations in exchange for Israel’s complete withdrawal from the territories occupied during the 1967 Six-Day War, the creation of a Palestinian state with East Jerusalem as its capital, and a resolution for Palestinian refugees based on UN General Assembly Resolution 194. While Israel has not officially signed the Arab Peace Initiative, it has been the subject of discussions and negotiations between Israel and the Arab states.

In summary, statement 1 is correct, and statement 2 is incorrect.

95. Consider the following pairs with regard to sports awards:

1. Major Dhyan Chand Khel Ratna Award : For the most spectacular and outstanding per- formance by a sportsperson over period of last four years.

2. Arjuna Award : For the lifetime achievement by a sportsperson

3. Dronacharya Award : To honour eminent coaches who have successfully trained sportspersons or teams.

4. Rashtriya Khel Protsahan Puraskar : To recognize the contribution made by sportspersons even after their retirement

How many of the above pairs are correctly matched?

(a) Only one
(b) Only two
(c) Only three
(d) All four

The correct answer is B, i.e Only two.

Pair 1 is correctly matched: The Khel Ratna Award is the highest sporting honor of India. It is awarded annually by the Ministry of Youth Affairs and Sports, Government of India. The recipient(s) are selected by a committee constituted by the Ministry and are honored for their “spectacular and most outstanding performance in the field of sports over a period of four years” at the international level. As of 2020, the award comprises a medallion, a certificate, and a cash prize of ₹25 lakh.

Pair 2 is incorrectly matched: The Arjuna Award, officially known as Arjuna Awards for Outstanding Performance in Sports and Games, is not the second-highest sporting honor of India. The recipients are selected by a committee constituted by the Ministry and are honored for their “good performance in the field of sports over a period of four years” at the international level and for having shown “qualities of leadership, sportsmanship, and a sense of discipline.”

Pair 3 is correctly matched: The Dronacharya Award, officially known as the Dronacharya Award for Outstanding Coaches in Sports and Games, is a sports coaching honor of the Republic of India. The award is named after Drona, often referred to as “Dronacharya” or “Guru Drona,” a character from the Sanskrit epic Mahabharata of ancient India. Recipients are selected by a committee constituted by the Ministry and are honored for having done “outstanding and meritorious work on a consistent basis and enabled sportspersons to excel in international events” over a period of four years.

Pair 4 is incorrectly matched: The Rashtriya Khel Protsahan Puruskar is not a sports honor of India. Instead, it is an award that recognizes the “involvement of corporates, voluntary organizations, and sports control boards in the promotion and development of sports in the country” over the previous three years.

96. Consider the following statements in respect of the 44th Chess Olympiad, 2022:

  1. It was the first time that Chess Olympiad was held in India.
  2. The official mascot was named Thambi’.
  3. The trophy for the winning team in the open section is the Vera Menchik Cup.
  4. The trophy for the winning team in the women’s section is the Hamilton-Russell Cup.

How many of the statements given above are correct?

(a) Only one
(b) Only two
(c) Only three
(d) All four

Answer – The correct answer is B, i.e Only two.

Statement 1 is correct: For the first time ever, India is hosting the world’s biggest chess event, the 44th Chess Olympiad, in Chennai.

Statement 2 is correct: The official mascot of the 44th Chess Olympiad is indeed ‘Thambi,‘ which means “little” or “younger brother” in the Tamil language.

Statement 3 and 4 are incorrect: The 44th Chess Olympiad witnessed the participation of 188 teams, representing 186 national federations in the open section, competing for the Hamilton-Russell Cup. In the women’s section, 162 teams from 160 federations vied for the championship, playing for the Vera Menchik Cup.

Source: Chess.com – 2022 FIDE Chess Olympiad

97. Consider the following pairs:

Area of conflict mentioned in news : Country where it is located

  1. Donbas – Syria
  2. Kachin – Ethiopia
  3. Tigray – North Yemen

How many of the above pairs are correctly matched?

(a) Only one
(b) Only two
(c) All three
(d) None

Answer – The correct answer is D, i.e None.

  • The Donbas or Donbass is a historical, cultural, and economic region in eastern Ukraine, so pair 1 is correctly matched.
  • Kachin State, also known by the endonym Kachinland, is indeed the northernmost state of Myanmar, so pair 2 is correctly matched.
  • Tigray is Ethiopia’s northernmost region, bordering Eritrea and home to a significant population of ethnic Tigrayans, so pair 3 is also correctly matched.

98. In the recent years Chad, Guinea, Mali and Sudan caught the international attention for which one of the following reasons common to all of them?

(a) Discovery of rich deposits of rare earth elements
(b) Establishment of Chinese military bases
(c) Southward expansion of Sahara Desert
(d) Successful coups

Answer – The correct answer is D, i.e Successful Coups.

In the past few years, several countries including Chad, Mali, Guinea, and Sudan have witnessed coup attempts or successful coups. Within the last 18 months, a similar pattern has emerged, as military leaders have overthrown the governments of Mali, Chad, Guinea, Sudan, and most recently, Burkina Faso.

99. Consider the following heavy industries

  1. Fertilizer plants.
  2. Oil refineries
  3. Steel plants

Green hydrogen is expected to play a significant role in decarbonizing how many of the above industries?

(a) Only one
(b) Only two
(c) All three
(d) None

Answer – The correct answer is C, i.e All three.

Green hydrogen has the potential to play a crucial role in decarbonizing various heavy industries, including fertilizer plants, oil refineries, and steel plants.

  1. Fertilizer Plants: Fertilizer production is known for its energy-intensive nature and reliance on fossil fuels, primarily natural gas. By adopting green hydrogen as a clean energy source, fertilizer plants can significantly reduce their carbon emissions. Green hydrogen can be employed to generate electricity or directly used as a feedstock in ammonia production, which is a vital component in fertilizer manufacturing.
  2. Oil Refineries: Oil refineries are major contributors to greenhouse gas emissions due to their energy-intensive processes and reliance on fossil fuels. Incorporating green hydrogen in oil refineries can help replace or supplement fossil fuels, thereby reducing their carbon footprint. Hydrogen can be utilized in refining processes such as hydrocracking and hydrotreating to eliminate impurities and enhance fuel quality.
  3. Steel Plants: The steel industry is a significant source of global carbon emissions. Green hydrogen can be utilized as a reducing agent in the steelmaking process, replacing the usage of coal or coke. This process, known as direct reduction, enables the production of “green” or low-carbon steel. Moreover, green hydrogen can be employed in the steel industry for heat generation and electricity production, leading to further reduction in carbon emissions.

100. Consider the following statements about G-20:

  1. The G-20 group was originally established as a platform for the Finance Ministers and Central Bank Governors to discuss the international economic and financial issues.
  2. Digital public infrastructure is one of India’s G-20 priorities.

Which of the statements given above is/are correct?

(a) 1 only
(b) 2 only
(c) Both 1 and 2
(d) Neither 1 nor 2

Answer – The correct answer is C, i.e Both 1 and 2.

Statement 1 is correct: The G20 was established in 1999 following the Asian financial crisis as a platform for Finance Ministers and Central Bank Governors to engage in discussions on global economic and financial matters.

Statement 2 is correct: India is advocating its digital infrastructure as an open-access platform within the G20. As part of India’s G20 Presidency, Digital Public Infrastructure is a prominent focus area. It is an overarching theme across different Working Groups, encompassing the Digital Economy, Health, Education, and the Global Partnership for Financial Inclusion.

Source: https://www.g20.org/en/about-g20/

For all the Previous Year Papers Click Here

For Daily Current Affairs Click Here

Join our Official Telegram Channel HERE
Subscribe to our YouTube Channel HERE
Follow our Instagram ID HERE

Similar Posts

Leave a Reply

Your email address will not be published. Required fields are marked *